G03.1 Quadrilaterals

Download as pdf or txt
Download as pdf or txt
You are on page 1of 83

QUADRILATERALS

REVISION: 3835

21 JULY 2024
AZIZ MANVA
AZIZMANVA@GMAIL.COM

ALL RIGHTS RESERVED


Get all the files at: https://bit.ly/azizhandouts
Aziz Manva (azizmanva@gmail.com)

TABLE OF CONTENTS
2.2 Visual Techniques 44
TABLE OF CONTENTS ................................. 2
3. QUADRILATERALS................................ 57
1. MENSURATION ....................................... 3
3.1 Sum of Angles 57
1.1 Parallelogram 3 3.2 Trapezoids 59
1.2 Rhombus 7 3.3 Parallelograms 72
1.3 Kites 9 3.4 Rhombi 79
2. SQUARES AND RECTANGLES ............. 12 3.5 General Quadrilaterals 81
3.6 Further Topics 83
2.1 Area and Perimeter Basics 12

P a g e 2 | 83
Get all the files at: https://bit.ly/azizhandouts
Aziz Manva (azizmanva@gmail.com)

1. MENSURATION
1.1 Parallelogram
A. Definition

1.1: (Def) Parallelogram


A parallelogram is a quadrilateral with opposite sides parallel.

➢ Special cases of parallelograms include squares, rectangles, and rhombi.

1.2: Special Trapezoid


A parallelogram is a special case of a trapezoid, since a trapezoid has at least one pair of parallel sides, and a
parallelogram has both pairs of sides parallel.

1.3: Properties of a Trapezoid/Quadrilateral.


Since a parallelogram is a
➢ special trapezoid, it inherits all the properties of a trapezoid.
➢ special trapezoid, it inherits all the properties of a quadrilateral.

B. Perimeter

Example 1.4
A parallelogram has adjacent sides with a total length of 12 cm. One pair of its opposite sides have a length of 10
cm. Find
A. The perimeter
B. Length of each side

The perimeter
2 × 𝐴𝑑𝑗𝑎𝑐𝑒𝑛𝑡 𝑆𝑖𝑑𝑒𝑠 = 2 × 12 = 24 𝑐𝑚

Since a pair of opposite sides = 10 cm hence:


𝑂𝑛𝑒 𝑜𝑓 𝑡ℎ𝑒𝑚 = 5 𝑐𝑚

Adjacent Sides are


5 𝑐𝑚 𝑎𝑛𝑑 12 – 5 = 7 𝑐𝑚
Sides are:
5 𝑐𝑚, 7𝑐𝑚, 5 𝑐𝑚, 𝑎𝑛𝑑 7 𝑐𝑚
C. Height

1.5: (Def) Height of a Parallelogram


The height of a parallelogram is the distance between the opposite sides.

➢ The height is often convenient to calculate when originating from a vertex, but this is not required.

P a g e 3 | 83
Get all the files at: https://bit.ly/azizhandouts
Aziz Manva (azizmanva@gmail.com)

Example 1.6
In parallelogram ABCD, draw:
A. the height from AB to CD
B. the height from AD to BC

In parallelogram ABCD, the height from AB to CD


= 𝐷𝑖𝑠𝑡𝑎𝑛𝑐𝑒 𝑏𝑒𝑡𝑤𝑒𝑒𝑛 𝐴𝐵 𝑎𝑛𝑑 𝐶𝐷 = 𝐴𝑋 = 𝑌𝐶

The height from AD to BC is a little more difficult to visualize:


= 𝐷𝑖𝑠𝑡𝑎𝑛𝑐𝑒 𝑏𝑒𝑡𝑤𝑒𝑒𝑛 𝐴𝐷 𝑎𝑛𝑑 𝐵𝐶 = 𝐴𝐴′ = 𝐷𝐷 ′ = 𝑃𝑄

Note that
𝑃𝑄 𝑑𝑜𝑒𝑠 𝑛𝑜𝑡 𝑜𝑟𝑖𝑔𝑖𝑛𝑎𝑡𝑒 𝑓𝑟𝑜𝑚 𝑎 𝑣𝑒𝑟𝑡𝑒𝑥

D. Area

1.7: Area of a Parallelogram


The area of a parallelogram is
𝐵𝑎𝑠𝑒 × 𝐻𝑒𝑖𝑔ℎ𝑡

Example 1.8
The sides of a parallelogram, in order, are 4𝑚, 6𝑚, 4𝑚, and 6𝑚. The height from the longer side is 2𝑚.
Determine the area of the parallelogram and the height from the shorter side.

The area of the parallelogram


= 𝑏1 ℎ1 = 6(2) = 12 𝑚2

The area of the parallelogram is also:


𝑏2 ℎ2 = 4(ℎ2 ) = 12 ⇒ ℎ2 = 3 𝑚

Example 1.9
The area of the shaded region 𝐵𝐸𝐷𝐶 in parallelogram ABCD is (AMC 8
1989/15)

The area of parallelogram 𝐴𝐵𝐶𝐷 will be


𝐴 = ℎ𝑏 = 10 ∙ 8 = 80

P a g e 4 | 83
Get all the files at: https://bit.ly/azizhandouts
Aziz Manva (azizmanva@gmail.com)

The area of Δ𝐴𝐸𝐵


1 1 1 1
= ℎ𝑏 = 𝐴𝐸 ∙ 𝐵𝐸 = (𝐴𝐷 − 𝐸𝐷)𝐵𝐸 = (10 − 6)8 = 16
2 2 2 2

The area of the shaded region

= [𝐴𝐵𝐶𝐷] − [𝐴𝐸𝐵] = 80 − 16 = 64

Example 1.10
The area in square units of the region enclosed by parallelogram 𝐴𝐵𝐶𝐷 is:
(AMC 8 1991/10)

The area of parallelogram 𝐴𝐵𝐶𝐷 will be


𝐴 = ℎ𝑏 = 2(4) = 8

E. Double Calculation of Area

1.11: Double Calculation of Area


The area of a parallelogram can be calculated in any way that we want and it will be the same always. In
particular:
𝐴 = ℎ1 𝑏1 = ℎ2 𝑏2

If you know any three out of ℎ1 , 𝑏1 , ℎ2, and 𝑏2 , then you can find the fourth quantity.

Example 1.12
Determine the number of values of 𝑥 such that the adjacent sides of a parallelogram are 𝑥 + 3 and 𝑥 + 4. And
the altitudes of the parallelogram have length 𝑥 + 5 and 𝑥 + 6.

We do not know which has which altitude. Hence, we consider cases:

Case I
(𝑥 + 3)(𝑥 + 6) = (𝑥 + 4)(𝑥 + 5)
𝑥 2 + 9𝑥 + 18 = 𝑥 2 + 9𝑥 + 20
18 = 20
𝑁𝑜 𝑆𝑜𝑙𝑢𝑡𝑖𝑜𝑛𝑠

Case IU
(𝑥 + 3)(𝑥 + 5) = (𝑥 + 4)(𝑥 + 6)
𝑥 2 + 8𝑥 + 15 = 𝑥 2 + 10𝑥 + 240
𝑥 = −4.5
𝑥 + 3 = −4.5 + 3 = −1.5 ⇒ 𝑁𝑜𝑡 𝑉𝑎𝑙𝑖𝑑
𝑁𝑜 𝑆𝑜𝑙𝑢𝑡𝑖𝑜𝑛𝑠

Hence, there is no value of 𝑥 that satisfies the above conditions.

P a g e 5 | 83
Get all the files at: https://bit.ly/azizhandouts
Aziz Manva (azizmanva@gmail.com)

Example 1.13
In parallelogram 𝐴𝐵𝐶𝐷, 𝐷𝐸 is the altitude to the
base 𝐴𝐵 and 𝐷𝐹 is the altitude to the base 𝐵𝐶.
[Note: Both pictures represent the same
parallelogram.] If 𝐷𝐶 = 12, 𝐸𝐵 = 4, and 𝐷𝐸 = 6,
then 𝐷𝐹 = (AMC 8 1995/24)

The opposite sides of a parallelogram are equal.


𝐴𝐵 = 𝐷𝐶

Substitute 𝐴𝐵 = 𝐴𝐸 + 𝐸𝐵 = 𝐴𝐸 + 4, 𝐷𝐶 = 12
𝐴𝐸 + 4 = 12 ⇒ 𝐴𝐸 = 8

DE is the altitude to the base AB. Therefore:


𝐷𝐸 ⊥ 𝐴𝐵 ⇒ Δ𝐷𝐴𝐸 𝑖𝑠 𝑎 𝑟𝑖𝑔ℎ𝑡 𝑡𝑟𝑖𝑎𝑛𝑔𝑙𝑒

In right triangle 𝐷𝐴𝐸, by Pythagorean Triplet:


(𝐷𝐴, 𝐷𝐸, 𝐴𝐸) = (10,6,8) ⇒ 𝐷𝐴 = 10

Since the opposite sides of a parallelogram are equal:


𝐵𝐶 = 10

Calculate the area of parallelogram 𝐴𝐵𝐶𝐷 in two different ways:


ℎ𝑏 = 𝐷𝐹 ∙ 𝐵𝐶 = 𝐷𝐸 ∙ 𝐴𝐵

Substitute 𝐷𝐸 = 6, 𝐴𝐵 = 12, 𝐵𝐶 = 10:


𝐷𝐹 ∙ 10 = 6 ∙ 12 = 72 ⇒ 𝐷𝐹 = 7.2

Example 1.14
A non-rectangular road runs parallel to a straight railway track. The sidewalks of the road are 10 feet apart. The
road is to be paved. What is the area to be paved if the road has a length of 20 feet.

The area of the road


= 𝐵𝑎𝑠𝑒 × 𝐻𝑒𝑖𝑔ℎ𝑡 = 20 ∙ 10 = 200 𝑓𝑡 2

Example 1.15
A street has parallel curbs 40 feet apart. A crosswalk bounded by two parallel stripes crosses the street at an
angle. The length of the curb between the stripes is 15 feet and each stripe is 50 feet long. Find the distance, in
feet, between the stripes? (AMC 10 2001/15)

P a g e 6 | 83
Get all the files at: https://bit.ly/azizhandouts
Aziz Manva (azizmanva@gmail.com)

Calculate the area of the parallelogram in two different ways:


ℎ(50) = 40(15)
ℎ = 12

1.2 Rhombus
A. Definition and Perimeter

1.16: (Def) Rhombus


A rhombus is a quadrilateral that all sides equal.

➢ A rhombus is a special case of a parallelogram.


➢ Since it is a parallelogram, it is also a trapezoid.

1.17: (Def) Perimeter


The perimeter of a rhombus is four times any of the sides.
𝑃𝑒𝑟𝑖𝑚𝑒𝑡𝑒𝑟 = 4𝑠, 𝑠 = 𝑆𝑖𝑑𝑒

Example 1.18
A rhombus has two sides which are 3𝑥 + 4 and 2𝑥 + 7. Determine the perimeter.

All sides of a rhombus are equal. Hence:


3𝑥 + 4 = 2𝑥 + 7
𝑥=3

Hence, one side is


3𝑥 + 4 = 3(3) + 4 = 9 + 4 = 13

The perimeter is
4𝑠 = 4(13) = 52

Example 1.19
What is the perimeter of a rhombus with
A. one side one foot, a second side 12 inches, and a third side 30.48 cm.
1
B. one of whose sides is 2 units long?
2+ 3
3+
4

Part A
Rhombuses have all sides equal
1 foot = 12 inches = (12 * 2.54) cm = 30.48 cm
P = 4 * 1 foot = 4 feet

Part B

P a g e 7 | 83
Get all the files at: https://bit.ly/azizhandouts
Aziz Manva (azizmanva@gmail.com)

1 4 4 30
4× = = =
2 4 38 19
2+ 3 2+2×
15 15
3+ 4

B. Area

1.20: Area (Diagonals)


If the diagonals of a rhombus have length 𝑑1 and 𝑑2 , then the area of the rhombus is half the product of the
diagonals:
𝑑1 𝑑2
𝐴𝑟𝑒𝑎 =
2

Example 1.21
What is the area of a rhombus with diagonals 4 cm?

1 1
𝐴𝑟𝑒𝑎 = 𝑑1 𝑑2 = (4)(4) = 8 𝑐𝑚2
2 2

Example 1.22
The area of a rhombus is 3 𝑓𝑡 2 . And the length of one of its diagonals is 3 𝑓𝑡. Determine the length of the other
diagonal.

𝑑1 𝑑2 3𝑑2
𝐴𝑟𝑒𝑎 = = =3
2 2
𝑑2 = 2

1.23: Diagonals of a Rhombus


Diagonals of a rhombus are perpendicular bisectors of each other.

Example 1.24S
Quadrilateral 𝐴𝐵𝐶𝐷 is a rhombus with perimeter 52 meters. The length of diagonal
𝐴𝐶 is 24 meters. What is the area in square meters of rhombus 𝐴𝐵𝐶𝐷? (AMC 8
2019/4)

Area
10 ∙ 24
= = 120
2

1.25: Area (Base and Height)


Since a rhombus is a parallelogram, the formula for area of a parallelogram is also applicable:
𝐴𝑟𝑒𝑎 = 𝐻𝑒𝑖𝑔ℎ𝑡 × 𝐵𝑎𝑠𝑒

Example 1.26
A rhombus has an area of 108 square units. The lengths of its diagonals have a ratio of 3 to 2. What is the length

P a g e 8 | 83
Get all the files at: https://bit.ly/azizhandouts
Aziz Manva (azizmanva@gmail.com)

of the longest diagonal, in units? (MathCounts 2007 State Countdown)

Write the given ratio, and use it to form a relation between the diagonals:
3: 2 = 3𝑥: 2𝑥

Create and simplify an expression for the area of the rhombus using the property that the area of a rhombus is
half the product of its diagonals:
3𝑥 × 2𝑥
𝐴= = 3𝑥 2
2

Equate this expression to the given value of the area, and solve the resulting equation:
3𝑥 2 = 108 ⇒ 𝑥 2 = 36 ⇒ 𝑥 = 6 ⇒ 3𝑥 = 18

Example 1.27
Rhombus 𝐴𝐵𝐶𝐷 is similar to rhombus 𝐵𝐹𝐷𝐸. The area of rhombus 𝐴𝐵𝐶𝐷 is 24 and ∠𝐵𝐴𝐷 = 60∘ . What is the
area of rhombus 𝐵𝐹𝐷𝐸? (AMC 10B 2006/15)

1.3 Kites
A. Definition and Perimeter

1.28: Definition
A kite has two pairs of adjacent equal sides.

In the diagram:
𝐴𝐵 = 𝐵𝐶, 𝐴𝐷 = 𝐷𝐶

1.29: Perimeter
𝑃 = 2(𝑠 + 𝑙)
Where
𝑠 = 𝑆ℎ𝑜𝑟𝑡𝑒𝑟 𝑠𝑖𝑑𝑒
𝑙 = 𝑙𝑜𝑛𝑔𝑒𝑟 𝑠𝑖𝑑𝑒

2(𝐴𝐵 + 𝐴𝐷)

Example 1.30
Find the perimeter of a kite:
A. with shorter side 0.25 m and longer side 20% longer than the shorter side.
3
B. where the longer side is 20% more than the shorter side, and the shorter side is 1
2− 3
1+
4

P a g e 9 | 83
Get all the files at: https://bit.ly/azizhandouts
Aziz Manva (azizmanva@gmail.com)

Part A
1 1 5 1 5 1 9 9
𝑃 =2× +2× × = 2 × (1 + ) = × =

4 ⏟
4 4 4 4 2 4 8
𝑆ℎ𝑜𝑟𝑡𝑒𝑟 𝑆𝑖𝑑𝑒 𝐿𝑜𝑛𝑔𝑒𝑟 𝑆𝑖𝑑𝑒

Part B
The shorter side
3 3 3 3 3 7 21
𝑠= = = = = =3× =
1 4 4 4 10 10 10
2− 3 1− 7 2−1× 2−
7 7 7
1+ 4 4
6𝑠 22𝑠 22 21 241
𝑃 = 2(𝑠 + 𝑙) = 2 (𝑠 + )= = × =

5 5 5 10 25
20% 𝑚𝑜𝑟𝑒

1.31: Diagonals of a Kite


Diagonals of a kite are perpendicular.

1.32: Area of a Kite


The area of a kite is half the product of the diagonals.
1
𝐴𝑟𝑒𝑎 = 𝑑1 𝑑2
2

The area of the kite is the sum of the two triangles that make up the kite:
[𝐴𝐵𝐶𝐷] = [𝐵𝐴𝐷] + [𝐵𝐶𝐷]

Using the formula for the area of a triangle:


1 1
= (𝐴𝑃)(𝐵𝐷) + (𝐶𝑃)(𝐵𝐷)
2 2
1
= (𝐵𝐷)[(𝐴𝑃) + (𝐶𝑃)]
2
1
= (𝐵𝐷)(𝐴𝐶)
2

Substitute 𝐵𝐷 = 𝑑1 , 𝐴𝐶 = 𝑑2
1
= 𝑑1 𝑑2
2

P a g e 10 | 83
Get all the files at: https://bit.ly/azizhandouts
Aziz Manva (azizmanva@gmail.com)

Example 1.33
𝑃𝑎𝑟𝑎𝑔𝑟𝑎𝑝ℎ 𝐵𝑎𝑠𝑒𝑑 𝑄𝑢𝑒𝑠𝑡𝑖𝑜𝑛
To promote her school's annual Kite Olympics, Genevieve makes a small kite and a
large kite for a bulletin board display. The kites look like the one in the diagram
below. For her small kite Genevieve draws the kite on a one-inch grid. For the large
kite she triples both the height and width of the entire grid.
A. What is the number of square inches in the area of the small kite? (AMC 8
2001/7)
B. Genevieve puts bracing on her large kite in the form of a cross connecting
opposite corners of the kite. How many inches of bracing material does she
need? (AMC 8 2001/8)
C. The large kite is covered with gold foil. The foil is cut from a rectangular piece that just covers the entire
grid. How many square inches of waste material are cut off from the four corners? (AMC 8 2001/9)

Part A
1 1
= 𝑑1 𝑑2 = (6)(7) = 21 𝑖𝑛2
2 2

Part B
The bracing material needed is equal to the length of the diagonals.
𝐿𝑒𝑛𝑔𝑡ℎ 𝑜𝑓 𝐷𝑖𝑎𝑔𝑜𝑛𝑎𝑙𝑠 = 𝐷1 + 𝐷2 = 3(6) + 3(7) = 18 + 21 = 39 𝑖𝑛𝑐ℎ𝑒𝑠

Part C
Area of the grid:
= 𝑙𝑤 = [3(7)][3(6)] = 9(42)

Area of the large kite


1 1 1
= 𝐷1 𝐷2 = [3(7)][3(6)] = ∙ 9(42)
2 2 2

Area of the waste material


1 9(42)
= 9(42) − ∙ 9(42) = = 9(21) = 189 𝑖𝑛2
2 2

Example 1.34
A kite is inscribed in a rectangle. Find the ratio of the area of the kite to the area of the rectangle.

The ratio that we want is:


1
𝐴𝑟𝑒𝑎 𝑜𝑓 𝐾𝑖𝑡𝑒 𝑑1 𝑑2
=2
𝐴𝑟𝑒𝑎 𝑜𝑓 𝑅𝑒𝑐𝑡𝑎𝑛𝑔𝑙𝑒 𝑙𝑤

But note that 𝑑1 = 𝑑2 = 𝑤:


1
𝑙𝑤 1
=2 =
𝑙𝑤 2

P a g e 11 | 83
Get all the files at: https://bit.ly/azizhandouts
Aziz Manva (azizmanva@gmail.com)

2. SQUARES AND RECTANGLES


2.1 Area and Perimeter Basics
A. Perimeter

2.1: Rectangle
For a rectangle with length 𝑙 and width 𝑤:
𝑃𝑒𝑟𝑖𝑚𝑒𝑡𝑒𝑟 = 2(𝑙 + 𝑤) = 2𝑙 + 2𝑤
𝐴𝑟𝑒𝑎 = 𝑙𝑤

2.2: Square
For a square with side length 𝑠
𝑃𝑒𝑟𝑖𝑚𝑒𝑡𝑒𝑟 = 4𝑠
𝐴𝑟𝑒𝑎 = 𝑠 2

Example 2.3
A. A square has side 3.5 units, while a rectangle has length 2.25 units and breadth 5.25 units. What is the
sum of their perimeters?
B. What is the perimeter of a square with area 64 units?
C. What is the perimeter of a rectangle with area 56 units, and one side 4 units?
D. A square and a triangle have equal perimeters. The lengths of the three sides of the triangle are 6.2 𝑐𝑚,
8.3 cm and 9.5 cm. The area of the square is: (AMC 8 1985/12)

Part A
3.5 × 4 + 2(2.25 + 5.25) = 14 + 2 × 7.5 = 14 + 15 = 29
Part B
Perimeter = Side × 4 = √64 × 4 = 8 × 4 = 32
Part C
The other side of the rectangle
56
= = 14
4
The perimeter
= 2(14 + 4) = 2 × 18 = 36
Part D
Perimeter of triangle
= 6.2 + 8.3 + 9.5 = 24
Side of square
24
= =6
4
Area of square
= 62 = 36

Example 2.4
A square with side 𝑠 is divided into five congruent rectangles stacked on top of each other. If the perimeter of
each rectangle is 1 foot, how many inches is the perimeter of the square?

P a g e 12 | 83
Get all the files at: https://bit.ly/azizhandouts
Aziz Manva (azizmanva@gmail.com)

𝑠
Substitute 𝑙 = 𝑠, 𝑤 = 5 in 𝑃 = 2(𝑙 + 𝑤):
𝑠
2 (𝑠 + ) = 12 𝑖𝑛𝑐ℎ𝑒𝑠
5
12𝑠
= 12
5
The side length
=𝑠=5

The perimeter of the square is:


4𝑠 = 20

Example 2.5
A square playground has a grassy area that is used to play football. The grassy
area occupies half of the area of the playground. The perimeter of the grassy area
is 100 𝑚𝑒𝑡𝑒𝑟𝑠. Find the perimeter and area of the entire playground.

The perimeter of the playground is:


𝑠 𝑠
𝑠 + 𝑠 + + = 100
2 2
3𝑠 = 100
100
𝑠=
3

The perimeter of the entire playground


400
= 4𝑠 = 𝑚
3

The area of the entire playground


2
100 2 10,000 2
=𝑠 =( ) = 𝑚
3 9

Example 2.6
Figures 𝐼, 𝐼𝐼, and 𝐼𝐼𝐼 are squares. The perimeter of 𝐼 is 12 and the perimeter of
𝐼𝐼 is 24. The perimeter of 𝐼𝐼𝐼 is (AMC 8 1995/6)

12 24 36
𝑃 = 4𝑠𝐼𝐼𝐼 = 4(𝑠𝐼 + 𝑠𝐼𝐼 ) = 4 ( + ) = 4 ( ) = 36
4 4 4

Example 2.7: TSD


Betty and Ann are walking around a rectangular park with dimensions 600 m by 400 m,
as shown. They both begin at the top left corner of the park and walk at constant but different speeds. Betty

P a g e 13 | 83
Get all the files at: https://bit.ly/azizhandouts
Aziz Manva (azizmanva@gmail.com)

walks in a clockwise direction and Ann walks in a counterclockwise direction. Points P, Q,R,S, T divide the
bottom edge of the park into six segments of equal length. When Betty and Ann meet for the first time, they are
between Q and R. Which of the following could be the ratio of Betty's speed to Ann's speed? (Gauss Grade 7
2017/20)
A. 5:3
B. 9:4
C. 11:6
D. 12:5
E. 17:7

Ann's distance:
𝑀𝑖𝑛: 600, 𝑀𝑎𝑥: 700

Betty' distance:
𝑀𝑖𝑛: 1300, 𝑀𝑎𝑥: 1400

Distance Ratios as Fractions:


𝑀𝑖𝑛 𝑜𝑓 𝐵𝑒𝑡𝑡𝑦 13 6
= =1
𝑀𝑎𝑥 𝑜𝑓 𝐴𝑛𝑛 7 7
𝑀𝑎𝑥 𝑜𝑓 𝐵𝑒𝑡𝑡𝑦 14 7 1
= = =2
𝑀𝑖𝑛 𝑜𝑓 𝐴𝑛𝑛 6 3 3

5 2 6
𝑂𝑝𝑡𝑖𝑜𝑛 𝐴: = 1 < 1
3 3 7
9 1 6 1 1
𝑂𝑝𝑡𝑖𝑜𝑛 𝐵: = 2 ⇒ 1 < 2 < 2 ⇒ 𝐶𝑜𝑟𝑟𝑒𝑐𝑡
4 4 7 4 3

Example 2.8
In the diagram, all angles are right angles and the lengths of the sides are
given in centimeters. Note the diagram is not drawn to scale. What is the
length in 𝑋, in centimeters? (AMC 8 2012/5)

2+𝑋 =6+1
𝑋 = 5 𝑐𝑚

Example 2.9
The letter 𝑇 is formed by placing two 2 × 4 inch rectangles next to each other, as shown. What is
the perimeter of the 𝑇, in inches? (AMC 8 2006/6)

P a g e 14 | 83
Get all the files at: https://bit.ly/azizhandouts
Aziz Manva (azizmanva@gmail.com)

Method I
Add the sides to get the perimeter
= 2 + 4 + 2 + 1 + 4 + 2 + 4 + 1 = 20

Method II
Note that the red line is missing from the perimeter of the shape. If the red line had been
a part of the perimeter, then the perimeter would have been:

2 × ⏟
2 × ⏟(4 + 2) = 4 × 6 = 24
𝐷𝑜𝑢𝑏𝑙𝑒 𝑓𝑜𝑟 𝐷𝑜𝑢𝑏𝑙𝑒 𝑓𝑜𝑟 𝐿𝑒𝑛𝑔𝑡ℎ 𝑝𝑙𝑢𝑠
𝑇𝑤𝑜 𝑅𝑒𝑐𝑡𝑎𝑛𝑔𝑙𝑒𝑠 𝑡ℎ𝑒 𝑆𝑖𝑑𝑒𝑠 𝑊𝑖𝑑𝑡ℎ

However, the red line must be subtracted to give a final perimeter of:
24 − 2 × 2 = 24 − 4 = 20

Example 2.10
Farmer Jane has a square grazing field with area 64 square units. She takes the fence from the field, and uses all
of it to fence a rectangular area with length 4 units. What is the percentage change in the grazing area??

𝑆𝑖𝑑𝑒 𝐿𝑒𝑛𝑔𝑡ℎ(𝑆𝑞𝑢𝑎𝑟𝑒) = √64 = 8


𝑃𝑒𝑟𝑖𝑚𝑒𝑡𝑒𝑟(𝑆𝑞𝑢𝑎𝑟𝑒) = 𝑃𝑒𝑟𝑖𝑚𝑒𝑡𝑒𝑟(𝑅𝑒𝑐𝑡𝑎𝑛𝑔𝑙𝑒) = 32

2(𝑙 + 𝑤) = 32
𝑙 + 𝑤 = 16
𝑤 = 16 − 𝑙 = 16 − 4 = 12

𝐴𝑟𝑒𝑎(𝑅𝑒𝑐𝑡𝑎𝑛𝑔𝑙𝑒) = (12)(4) = 48

64 − 48 16 1
% 𝑐ℎ𝑎𝑛𝑔𝑒 = = = = 25% 𝑟𝑒𝑑𝑢𝑐𝑡𝑖𝑜𝑛
64 64 4

Example 2.11
A rectangular garden 60 feet long and 20 feet wide is enclosed by a fence. To make the garden larger, while
using the same fence, its shape is changed to a square. By how many square feet does this enlarge the garden?
(AMC 8 1999/5)

𝑃𝑒𝑟𝑖𝑚𝑒𝑡𝑒𝑟(𝑅𝑒𝑐𝑡𝑎𝑛𝑔𝑙𝑒) = 2(60 + 20) = 2(80) = 160


160
𝑆𝑖𝑑𝑒(𝑆𝑞𝑢𝑎𝑟𝑒) = = 40
4
𝐴𝑟𝑒𝑎(𝑆𝑞𝑢𝑎𝑟𝑒) − 𝐴𝑟𝑒𝑎(𝑅𝑒𝑐𝑡𝑎𝑛𝑔𝑙𝑒) = 402 − (60)(20) = 1600 − 1200 = 400

Example 2.12
A rectangular grazing area is to be fenced off on three sides using part of a 100-meter rock wall as the fourth
side. Fence posts are to be placed every 12 meters along the fence including the two posts where the fence
meets the rock wall. What is the fewest number of posts required to fence an area 36 m by 60 m? (AMC 8
1986/18)

To minimize the number of posts, make the side with length 60 m parallel to the rock wall. Then
𝐿𝑒𝑛𝑔𝑡ℎ 𝑜𝑓 𝐹𝑒𝑛𝑐𝑒 36 + 60 + 36 132
𝑁𝑜. 𝑜𝑓 𝐹𝑒𝑛𝑐𝑒𝑝𝑜𝑠𝑡𝑠 = +1= +1 = + 1 = 11 + 1 = 12
𝐼𝑛𝑡𝑒𝑟𝑣𝑎𝑙 6 12

P a g e 15 | 83
Get all the files at: https://bit.ly/azizhandouts
Aziz Manva (azizmanva@gmail.com)

Example 2.13
Carl decided to fence in his rectangular garden. He bought 20 fence posts, placed one on each of the four
corners, and spaced out the rest evenly along the edges of the garden, leaving exactly 4 yards between
neighboring posts. The longer side of his garden, including the corners, has twice as many posts as the shorter
side, including the corners. What is the area, in square yards, of Carl’s Garden? (AMC 10B 2016/11)

Out of the 20 fenceposts, 4 are at the corners, leaving for the sides:
20 − 4 = 16
These 16 fenceposts are equally divided among the opposite sides:
2(𝐹𝐿𝑒𝑛𝑔𝑡ℎ + 𝐹𝑊𝑖𝑑𝑡ℎ ) = 16
𝐹𝐿𝑒𝑛𝑔𝑡ℎ + 𝐹𝑊𝑖𝑑𝑡ℎ = 8

Method I: Trial and Error


If the width has 1 fencepost not including the corners, then it has the length
has
8 − 1 = 7 𝑓𝑒𝑛𝑐𝑒𝑝𝑜𝑠𝑡𝑠
Including the corners, we have:
𝑊𝑖𝑑𝑡ℎ = 1 + 2 = 3
𝐿𝑒𝑛𝑔𝑡ℎ = 7 + 2 = 9 ≠ 2(3) = 6 ⇒ 𝑁𝑜𝑡 𝑉𝑎𝑙𝑖𝑑

If we increase the width to have 2 fenceposts, them including the corners,


we have
𝑊𝑖𝑑𝑡ℎ = 2 + 2 = 4
𝐿𝑒𝑛𝑔𝑡ℎ = 6 + 2 = 8 = 2(4) ⇒ 𝑉𝑎𝑙𝑖𝑑

Method II: Algebra


𝑤, 8 − 𝑤
The total number of fenceposts on the width is:
𝑤+2
The total number of fencepossts on the lenght is:
8 − 𝑤 + 2 = 10 + 𝑤
10 − 𝑤 = 2(𝑤 + 2)
𝑤=2

Finding the Area


Whichever method, we use, the area is:
𝐴𝑟𝑒𝑎 = 𝑙𝑤 = 4(3) ⏟ = (12)(28) = 336 𝑦𝑎𝑟𝑑𝑠 2
⏟ × 4(7)
𝑊𝑖𝑑𝑡ℎ 𝐿𝑒𝑛𝑔𝑡ℎ

B. Diagonals

2.14: Diagonals of a Rectangle


The diagonals of a rectangle have equal length.

Example 2.15
Milee went from the top left corner of a rectangular field to the bottom right
corner, and counted 38 paces. Milee’s friend takes paces which are half the size

P a g e 16 | 83
Get all the files at: https://bit.ly/azizhandouts
Aziz Manva (azizmanva@gmail.com)

of Milee’s. How many paces will she take to go from the bottom left corner to the top right corner?

𝑀𝑖𝑙𝑒𝑒 ′ 𝑠 𝑓𝑟𝑖𝑒𝑛𝑑 = 2(38) = 76 𝑝𝑎𝑐𝑒𝑠

2.16: Proving a Rectangle


If the diagonals of a parallelogram are congruent, then the parallelogram is a rectangle.

Example 2.17
Two sides of a parallelogram are 4 units and 5 units. The diagonals of the parallelogram are each √41 units.
Find the area of the parallelogram.

Since the diagonals are equal, the parallelogram is a rectangle. Hence:


𝐴 = 𝑙𝑤 = 4(5) = 20 𝑢𝑛𝑖𝑡𝑠 2

2.18: Diagonal of a Square


The diagonal (𝑑) of a square with side length 𝑠 is
𝑑 = √2 × 𝑠

Without loss of generality, let the length of the side of the square be 1.
Then, in right Δ𝐴𝐷𝐵, by Pythagoras Theorem:
𝐴𝐶 2 = 12 + 12 = 1 + 1 = 2

Take the square root both sides:


𝐴𝐶 = √2

Example 2.19
A. What is the length of the diagonal of a square of side 3 units?
B. A farmer has a square field with a side length of 100𝑚. Determine the length of the diagonal in 𝑘𝑚.

Part A
Length of the diagonal
= √2 × 𝑆𝑖𝑑𝑒 = √2 × 3 = 3√2 𝑢𝑛𝑖𝑡𝑠
Part B
1
𝑆𝑖𝑑𝑒 𝐿𝑒𝑛𝑔𝑡ℎ = 100𝑚 = 𝑘𝑚
10
1 √2
𝐷𝑖𝑎𝑔𝑜𝑛𝑎𝑙 = × √2 𝑘𝑚 = 𝑘𝑚
10 10

Example 2.20
A. A boy walks three times around his square playground, and takes 120 paces to do so. Determine how
many paces the boy will take to go from one corner to a non-adjacent corner. (Your answer need not be
an integer).
B. A square field has area 16 square units. What is the distance from the top left corner of the field to the
bottom right corner?

Part A
3𝑃 = 120 𝑃𝑎𝑐𝑒𝑠
𝑃 = 40 𝑝𝑎𝑐𝑒𝑠

P a g e 17 | 83
Get all the files at: https://bit.ly/azizhandouts
Aziz Manva (azizmanva@gmail.com)

𝑃
𝑠= = 10 𝑝𝑎𝑐𝑒𝑠
4
𝑑 = √2𝑠 = √2 × 10 = 10√2
Part B
The area of the square is
𝐴 = 𝑠 2 = 16
Then, the side length is:
𝑠 = √16 = 4
And the distance from top left to bottom right is the diagonal, which is:
𝑑 = 4 × √2 = 4√2

2.21: Side Length of a Square


The diagonal (𝑑) of a square with side length 𝑠 is
𝑑
𝑑 = √2 × 𝑠 ⇒ 𝑠 =
√2

Example 2.22
A. Find the perimeter of a square with diagonal 5√2 𝑚.
B. What is the length of the side of a square, the sum of whose diagonals is 12
units?
C. Half the length of the diagonal of a square is 5. Determine the perimeter of the square.

Part A
5√2
𝑠= =5
√2
𝑃 = 4𝑠 = 20
Part B
Since the sum of the diagonals is 12, first find the length of a single diagonal:
2 × 𝑑𝑖𝑎𝑔𝑜𝑛𝑎𝑙 = 12 ⇒ 𝐷𝑖𝑎𝑔𝑜𝑛𝑎𝑙 = 𝑑 = 6
The side length is:
6 6 √2 6√2
𝑠= = ∙ = = 3√2
√2 √2 √2 2
Part C
𝑑
= 5 ⇒ 𝑑 = 10
2
10 10 √2 10√2
𝑠= = ∙ = = 5√2
√2 √2 √2 2
𝑃 = 4𝑠 = 4(5√2) = 20√2

Example 2.23
Calculate the cost of fencing a square field which has a distance of 5𝑚 between its nonadjacent corners if the
cost of fencing 1𝑚 is √3 dollars.

The side length is:


𝑑 5 5 √2 5√2
𝑠= = = ∙ = 𝑚
√2 √2 √2 √2 2
The perimeter

P a g e 18 | 83
Get all the files at: https://bit.ly/azizhandouts
Aziz Manva (azizmanva@gmail.com)

5√2
𝑃 = 4𝑠 = 4 × = 10√2
2
$
𝑇𝑜𝑡𝑎𝑙 𝐶𝑜𝑠𝑡 = 𝑃𝑐 = (10√2𝑚) (√3 ) = 10√6 𝑑𝑜𝑙𝑙𝑎𝑟𝑠
𝑚

2.24: Diagonal of a Rectangle


Given a rectangle with length 𝑙 and breadth 𝑏, the length of the diagonal is
𝑑 = √𝑙 2 + 𝑏 2

Since 𝐴𝐵𝐶𝐷 is a rectangle


∠𝐴𝐵𝐶 = 90°
In right Δ𝐴𝐵𝐶, by Pythagoras Theorem:
𝑑2 = 𝐷𝑖𝑎𝑔𝑜𝑛𝑎𝑙 2 = 𝐴𝐶 2 = 𝐴𝐵2 + 𝐵𝐶 2 = 𝑙 2 + 𝑏 2
𝑑 = √𝑙 2 + 𝑏 2

Example 2.25
A. What is the length of the diagonal of a rectangle of length 15 units and breadth 8 units?
B. Find the length of the longest possible straight path in a rectangular park of perimeter 34, and length 5.

Part A
𝐷𝑖𝑎𝑔𝑜𝑛𝑎𝑙 = √𝑙 2 + 𝑏 2 = √152 + 82 = √225 + 64 = 17 𝑢𝑛𝑖𝑡𝑠
𝑂𝑅 𝑢𝑠𝑖𝑛𝑔 𝑃𝑦𝑡ℎ𝑎𝑔𝑜𝑟𝑒𝑎𝑛 𝑇𝑟𝑖𝑝𝑙𝑒𝑡 (8,15,17) ⇒ 𝐷𝑖𝑎𝑔𝑜𝑛𝑎𝑙 = 17
Part B
34 − 5 × 2 24
𝐵𝑟𝑒𝑎𝑑𝑡ℎ = = = 12
2 2
𝐷𝑖𝑎𝑔𝑜𝑛𝑎𝑙 = √52 + 122 = √169 = 13
𝑂𝑅 𝑢𝑠𝑖𝑛𝑔 𝑃𝑦𝑡ℎ𝑎𝑔𝑜𝑟𝑒𝑎𝑛 𝑇𝑟𝑖𝑝𝑙𝑒𝑡 (5,12,13) ⇒ 𝐷𝑖𝑎𝑔𝑜𝑛𝑎𝑙 = 13

2.26: Area of a Square in terms of its diagonal


The area of a square with diagonal 𝑑 is
1 2
𝑑
2

By Pythagoras Theorem in Δ𝐴𝐷𝐶


2 2 2
𝑑2 2
𝑠 +𝑠 =𝑑 ⇒𝑠 =
2

Example 2.27
It takes Paridhi twelve minutes to walk through the shortest path from one
vertex to a non-adjacent vertex of a square field while walking at the rate of
𝑘𝑚
6 . What is the area of the field, in square meters?
ℎ𝑟

P a g e 19 | 83
Get all the files at: https://bit.ly/azizhandouts
Aziz Manva (azizmanva@gmail.com)

Determine the length of the path using


𝑘𝑚 𝑘𝑚 1 6
𝐷𝑖𝑎𝑔𝑜𝑛𝑎𝑙 = 6 ∙⏟
12 𝑚𝑖𝑛 = 6 ∙ ℎ𝑟 = 𝑘𝑚 = 1.2 𝑘𝑚
⏟ ℎ𝑟 ℎ𝑟 5 5
𝑻𝒊𝒎𝒆
𝑺𝒑𝒆𝒆𝒅

The area of the field is:


1 1 1
𝐴 = 𝑑2 = ∙ 1.22 = ∙ 1.2 ∙ 1.2 = 0.6 ∙ 1.2 = 0.72 𝑘𝑚2
2 2 2
= 0.72 (𝑘𝑚)(𝑘𝑚)

Convert from 𝑘𝑚2 to 𝑚2 by multiplying 1000:


= 0.72 (1000𝑚)(1000𝑚) = 720000 𝑚2

Example 2.28
It takes Pari fifteen minutes to walk through the longest straight path from one vertex to a non-adjacent vertex
𝑘𝑚
of a square field. She walked at 4 ℎ𝑟
. Determine the cost of planting the field with grass at $0.25 per square
meter.

Determine the length of the path using


𝑘𝑚 𝑘𝑚 1
𝐷𝑖𝑎𝑔𝑜𝑛𝑎𝑙 = 4 ∙⏟
15 𝑚𝑖𝑛 = 4 ∙ ℎ𝑟 = 1 𝑘𝑚
⏟ℎ𝑟 ℎ𝑟 4
𝑻𝒊𝒎𝒆
𝑺𝒑𝒆𝒆𝒅

The area of the field is:


1 1 1 1
𝐴 = 𝑑2 = ∙ 12 = 𝑘𝑚2 = (𝑘𝑚)(𝑘𝑚)
2 2 2 2

Convert from 𝑘𝑚2 to 𝑚2 by writing 1 𝑘𝑚 = 1000𝑚:


1
= (1000𝑚)(1000𝑚) = 500,000 𝑚2
2

Calculate the cost:


1
= × 500,000 = 125,000$
4

2.29: Area of a Rectangle in terms of its diagonal


A rectangle 𝑐𝑎𝑛𝑛𝑜𝑡 be uniquely identified based on the length of its diagonal.
1
𝐴𝑟𝑒𝑎 ≠ 𝑑2
2

This is proved by way of a counterexample in the next question.

Example 2.30
Consider a rectangle 𝑅1 with dimensions (𝑏, 𝑙) = (15,20). Consider another rectangle 𝑅2 with dimensions(𝑏, 𝑙) =
(7,24). Show that the two rectangles have same diagonal, but different areas.

Using Pythagorean Triplets:


𝐷1 = (15,20) = 5(3,4,5) = (15,20,25)
𝐷2 = (7,24,25)

P a g e 20 | 83
Get all the files at: https://bit.ly/azizhandouts
Aziz Manva (azizmanva@gmail.com)

Calculate the area:


𝐴1 = 15 × 20 = 300
𝐴2 = 7 × 24 = 168

The diagonals are the same, but the areas are different.

Example 2.31
A rectangular parking lot has a diagonal of 25 meters and an area of 168 square meters. In meters, what is the
perimeter of the parking lot? (AMC 10B 2011/14)

Square both sides of √⬚ = 25


𝑙⏟2 + 𝑏 2 = 625
𝑬𝒒𝒖𝒂𝒕𝒊𝒐𝒏 𝑰

The area is:


𝑙𝑏 = 168 ⇒ 2𝐴 = ⏟
2𝑙𝑏 = 336
𝑬𝒒𝒖𝒂𝒕𝒊𝒐𝒏 𝑰𝑰

𝑙 2 + 2𝑙𝑏 + 𝑏 2 = 961
(𝑙 + 𝑏)2 = 312
𝑙 + 𝑏 = 31
𝑃 = 2(𝑙 + 𝑏) = 62

2.32: Midpoints form a Rectangle


The quadrilateral formed by joining the midpoints of the sides of a square with side length 2𝑠 in order forms a
square with side length
√2𝑠

Consider square ABCD with side length


2𝑠

It has midpoints 𝐸, 𝐹, 𝐺 and 𝐻:


𝐸𝐷 = 𝐷𝐻 = 𝑠

By Pythagoras Theorem in right Δ𝐸𝐷𝐻


𝐸𝐻 = √𝐸𝐷 2 + 𝐷𝐻 2 = √2𝑠 2 = √2𝑠

By similar logic:

𝐸𝐹 = 𝐹𝐺 = 𝐻𝐺 = √2𝑠

Example 2.33
A square 𝐴𝐵𝐶𝐷 has side length √13. 𝑃, 𝑄, 𝑅 and S are the midpoints of sides 𝐴𝐵, 𝐵𝐶, 𝐶𝐷 and 𝐷𝐴. Determine the
ratio of the area of quadrilateral 𝑃𝑄𝑅𝑆 to the area of quadrilateral 𝐴𝐵𝐶𝐷.

P a g e 21 | 83
Get all the files at: https://bit.ly/azizhandouts
Aziz Manva (azizmanva@gmail.com)

2
(√2𝑠) : (2𝑠)2 = 2𝑠 2 : 4𝑠 2 = 2: 4 = 1: 2

Example 2.34
Triangle 𝐴𝐵𝐶 is an isosceles triangle with 𝐴𝐵 = 𝐵𝐶. Point 𝐷 is the midpoint of both 𝐵𝐶
and 𝐴𝐸, and 𝐶𝐸 is 11 units long. Triangle 𝐴𝐵𝐷 is congruent to triangle 𝐸𝐶𝐷. What is the
length of 𝐵𝐷? (AMC 8 2006/19)

Example 2.35
A large square region is paved with 𝑛2 gray square tiles, each measuring 𝑠 inches
on a side. A border 𝑑 inches wide surrounds each tile. The figure shows the case for
𝑛 = 3. When 𝑛 = 24, the 576 gray tiles cover 64% of the area of the large square
𝑑
region. What is the ratio 𝑠 for this larger value of 𝑛? (AMC 8 2020/24)

Area of the gray square tiles


The total area of the grey tiles
⏟2 ∙
= 24 𝑠⏟2 = 242 ∙ 𝑠 2
𝑵𝒐. 𝒐𝒇 𝑨𝒓𝒆𝒂 𝒐𝒇
𝑻𝒊𝒍𝒆𝒔 𝒆𝒂𝒄𝒉 𝒕𝒊𝒍𝒆

Length of Square
For 3 tiles, the length of the square will be:
3𝑠 + 4𝑑
For 24 tiles, the total length of the gaps will be:
24𝑠 + 25𝑑

Area of Square
Therefore, the area of the entire square will be:
𝑆𝑖𝑑𝑒 2 = (24𝑠 + 25𝑑)2

64
The ratio of the grey tiles to the larger square is 64% =
100
242 ∙ 𝑠 2 64
=
(24𝑠 + 25𝑑)2 100

P a g e 22 | 83
Get all the files at: https://bit.ly/azizhandouts
Aziz Manva (azizmanva@gmail.com)

Take the square root both sides:


24𝑠 8 4
= =
24𝑠 + 25𝑑 10 5

Since the numerator and the denominator both have 24𝑠, take the reciprocal on both sides:
24𝑠 25𝑑 5
+ =
24𝑠 24𝑠 4
24𝑠
Substitute 24𝑠 = 1:
25𝑑 5 25𝑑 1 25𝑑 𝑑 6
1+ = ⇒ = ⇒ =1⇒ =
24𝑠 4 24𝑠 4 6𝑠 𝑠 25

C. Calculations

Example 2.36
Six rectangles each with a common base width of 2 have lengths of 1, 4, 9, 16, 25, and 36. What is the sum of the
areas of the six rectangles? (AMC 8 2014/6)

The area that we want is:


2 ∙ 1 + 2 ∙ 4 + 2 ∙ 9 + 2 ∙ 16 + 2 ∙ 25 + 2 ∙ 36

Factor 2:
2(1 + 4 + 9 + 16 + 25 + 36)

Note that
𝑛(𝑛 + 1)(2𝑛 + 1)
12 + 22 + ⋯ + 𝑛 2 =
6

Substituting 𝑛 = 6:
6 ∙ 7 ∙ 13
2∙ = 2(91) = 182
6

Example 2.37
Karl's rectangular vegetable garden is 20 feet by 45 feet, and Makenna's is 25 feet by 40 feet. Which of the
following statements are true?
(𝐴)Karl's garden is larger by 100 square feet.
(𝐵)Karl's garden is larger by 25 square feet.
(𝐶)The gardens are the same size.
(𝐷)Makenna's garden is larger by 25 square feet.
(𝐸)Makenna's garden is larger by 100 square feet. (AMC 8 2011/2)

𝐴𝑟𝑒𝑎𝐾𝑎𝑟𝑙 = 20(45) = 900


𝐴𝑟𝑒𝑎𝑀𝑎𝑘𝑒𝑛𝑛𝑎 = 25(40) = 1000

𝑂𝑝𝑡𝑖𝑜𝑛 𝐸

Example 2.38
How many square yards of carpet are required to cover a rectangular floor that is 12 feet long and 9 feet wide?

P a g e 23 | 83
Get all the files at: https://bit.ly/azizhandouts
Aziz Manva (azizmanva@gmail.com)

(There are 3 feet in a yard.) (AMC 8 2015/1)

Convert the dimensions from feet to yards, and then find the area:
12 9
× = 4 × 3 = 12
3 3

Shortcut
Find the area and then divide by 32 = 9 to convert to square yards:
12 × 9
= 12
32

Example 2.39
At a store, when a length is reported as 𝑥 inches that means the length is at least 𝑥 − 0.5 inches and at most 𝑥 +
0.5 inches. Suppose the dimensions of a rectangular tile are reported as 2 inches by 3 inches. In square inches,
what is the
A. minimum area for the rectangle? (AMC 10B 2011/3)
B. maximum area for the rectangle?

Part A
The minimum dimensions are:
3 5
2 − 0.5 = 1.5 = , 3 − 0.5 = 2.5 =
2 2
The area
3 5 15
= ∙ = = 3.75 𝑖𝑛2
2 2 4
Part B
The maximum dimensions are:
5 7
2 + 0.5 = 2.5 = , 3 + 0.5 = 3.5 =
2 2
The area
5 7 35
= ∙ = = 8.75 𝑖𝑛2
2 2 4

Example 2.40
Tyler is tiling the floor of his 12-foot by 16-foot living room. He plans to place one-foot by one-foot square tiles
to form a border along the edges of the room and to fill in the rest of the floor with two-foot by two-foot square
tiles. How many tiles will he use? (AMC 8 2018/9)

The area of the border will be:


12 × 16 − ⏟
⏟ 10 × 14 = 192 − 140 = 52
𝑇𝑜𝑡𝑎𝑙 𝐴𝑟𝑒𝑎 𝐼𝑛𝑛𝑒𝑟 𝐴𝑟𝑒𝑎

Hence, the number of border tiles


52
= = 52
1

The number of inner tiles


𝐼𝑛𝑛𝑒𝑟 𝐴𝑟𝑒𝑎 140
= = 35
𝐴𝑟𝑒𝑎 𝑜𝑓 𝑒𝑎𝑐ℎ 𝑡𝑖𝑙𝑒 2 × 2

P a g e 24 | 83
Get all the files at: https://bit.ly/azizhandouts
Aziz Manva (azizmanva@gmail.com)

The total tiles


= 52 + 35 = 87

Example 2.41
Carrie has a rectangular garden that measures 6 feet by 8 feet. She plants the entire garden with strawberry
plants. Carrie is able to plant 4 strawberry plants per square foot, and she harvests an average of 10
strawberries per plant. How many strawberries can she expect to harvest? (AMC 8 2020/3)

The area of the garden is:


6 × 8 = 48 𝑓𝑡 2

The number of strawberry plants is:


48 × 4 = 192

The number of strawberries


= 192 × 10 = 1920

Example 2.42
A rectangle with a diagonal of length 𝑥 is twice as long as it is wide. What is the area of the rectangle? (Answer
in terms of 𝑥) (AMC 10A 2005/4)

Draw the rectangle with width 𝑤 and length 2𝑤. It has area:
𝑤 × 2𝑤 = 2𝑤 2

By Pythagoras Theorem in right Δ𝐴𝐷𝐶:


𝑤 2 + (2𝑤)2 = 𝑥 2
𝑤 2 + 4𝑤 2 = 𝑥 2
5𝑤 2 = 𝑥 2
𝑥2
𝑤2 =
5
2
2𝑤 2 = 𝑥 2
5

Example 2.43
If four identical squares are stacked in a single row (such that the lowest square touches the bottom, and the
highest square touches the top) in a larger square with area 784 units, what is the area of the larger square not
covered by the smaller squares?

The side length of the larger square is


√784 = 28

The side length of the smaller square


28
= =7
4

The area of each smaller square


= 72 = 49

P a g e 25 | 83
Get all the files at: https://bit.ly/azizhandouts
Aziz Manva (azizmanva@gmail.com)

The area that we want is:


784 − 49 × 4 = 784 − 200 + 4 = 588

Example 2.44
The area of the rectangular region is (AMC 8 1987/5)

0.22 × 0.4 = 22 × 4 × 0.001 = 0.088 𝑚2

Example 2.45
The area of this figure is 100 cm2 . Its perimeter is (AMC 8 1990/15)
[figure consists of four identical squares]

The area of each square


100
= = 25 𝑐𝑚2
4

The side length of each square


= √25 = 5 𝑐𝑚

Method I
Since there are 10 sides (see diagram to the right), the perimeter
= 5 × 10 = 50 𝑐𝑚

Method II: Complementary Counting


If we had four complete squares, then they would have perimeter:
= ⏟ 5 × 4 × 4 = 80 𝑐𝑚
𝑃𝑒𝑟𝑖𝑚𝑒𝑡𝑒𝑟
𝑜𝑓 1 𝑠𝑞𝑢𝑎𝑟𝑒

But the dotted lines are not part of the perimeter. We have 3 dotted lines. But each dotted is a part of two
squares. Hence, we must count it twice:
𝐷𝑜𝑡𝑡𝑒𝑑 𝐿𝑖𝑛𝑒𝑠 = 2 × 3 = 6

Hence, the length missing is:


= 5 × 6 = 30 𝑐𝑚

The final answer is:


80 − 30 = 50 𝑐𝑚

Example 2.46
The rectangle shown has length 𝐴𝐶 = 32, width 𝐴𝐸 = 20, and 𝐵 and 𝐹 are
midpoints of 𝐴𝐶 and 𝐴𝐸, respectively. The area of quadrilateral 𝐴𝐵𝐷𝐹 is: (AMC 8
1993/18)

P a g e 26 | 83
Get all the files at: https://bit.ly/azizhandouts
Aziz Manva (azizmanva@gmail.com)

The area that we want is:


[𝐴𝐵𝐶𝐷] − [𝐵𝐶𝐷] − [𝐹𝐸𝐷]
1 1
= 32 × 20 − × 16 × 20 − × 10 × 32
2 2
= 32 × 20 − 16 × 10 − 10 × 16
= 640 − 160 − 160
= 320

Example 2.47
A checkerboard consists of one-inch squares. A square card, 1.5 inches on a side, is placed on the board so that
it covers part or all of the area of each of 𝑛 squares. The maximum possible value of 𝑛 is (AMC 8 1993/25)

Calculate the diagonal of the square card:


√(1.5)2 + (1.5)2 = √2.25 + 2.25 = √4.5 > √4 = 2

P a g e 27 | 83
Get all the files at: https://bit.ly/azizhandouts
Aziz Manva (azizmanva@gmail.com)

D. Triangles

Example 2.48: Diagonals


1
𝑃𝑄𝑅𝑆 is a square. A is the midpoint of side 𝑃𝑄. 𝐵 is a point on 𝑅𝑆 such that 𝑅𝐵 is 3 𝑟𝑑 of 𝑅𝑆. If the area of Δ𝐴𝑄𝐵
is 𝑡 units, find the length of the diagonal of the square in terms of 𝑡.

Let the side length of the square be


𝑠 ⇒ 𝐷𝑖𝑎𝑔𝑜𝑛𝑎𝑙 = √2𝑠
The area of Δ𝐴𝑄𝐵
1 1 1 1 1
= ℎ𝑏 = (𝑃𝑆)(𝐴𝑄) = 𝑠 ( 𝑠) = 𝑠 2
2 2 2 2 4

But the area of the triangle is 𝑡


1 2
𝑠 =𝑡
4
𝑠 2 = 4𝑡
𝑠 = 2√𝑡
√2𝑠 = 2√2𝑡

Example 2.49: Diagonals


2
𝑃𝑄𝑅𝑆 is a square. A is the midpoint of side 𝑃𝑄. 𝐵 is a point on 𝑅𝑄 such that 𝑅𝐵 is 3 𝑟𝑑 of 𝑅𝑄. If the area of ΔAQB
is 𝑡 units, find the length of the diagonal of the square in terms of 𝑡.

Let the length of side of the square


=𝑠

1 1 1 𝑠 𝑠 𝑠2
Δ𝐴𝑄𝐵 = ℎ𝑏 = (𝐴𝑄)(𝑄𝐵) = × 𝑠 × 𝑠 =
2 2 2 2 3 12
𝑠2
= 𝑡 ⇒ 𝑠 2 = 12𝑡 ⇒ 𝑠 = √12𝑡
12
𝐷𝑖𝑎𝑔𝑜𝑛𝑎𝑙 = √2 × √12𝑡 = √24𝑡 = √4 × √6𝑡 = 2√6𝑡

Example 2.50
Square 𝐴𝐵𝐶𝐷 has sides of length 3. Segments 𝐶𝑀 and 𝐶𝑁 divide the square's area
into three equal parts. How long is segment 𝐶𝑀? (AMC 8 1999/23)

The area of square 𝐴𝐵𝐶𝐷


= 32 = 9

The area of each triangle


9
= =3
3
The area of Δ𝐵𝐶𝑀

P a g e 28 | 83
Get all the files at: https://bit.ly/azizhandouts
Aziz Manva (azizmanva@gmail.com)

1 1
= (𝐵𝐶)(𝐵𝑀) = (3)(𝐵𝑀) = 3 ⇒ 𝐵𝑀 = 2
2 2

By Pythagoras Theorem in Δ𝐵𝐶𝑀:


𝐶𝑀 = √𝐵𝐶 2 + 𝐵𝑀2 = √32 + 22 = √9 + 4 = √13
E. Percentage Change

Example 2.51
Each side of a square is increased by 30%. The percentage increase in area is:

Consider a square of side length 𝑠. The side is increased by:


30% 𝑜𝑓 𝑠 = 0.3𝑠

New Area will be


(𝑠 + 0.3𝑠)2 = (1.3𝑠)2 = 1.69𝑠 2

The percentage increase is:


𝐼𝑛𝑐𝑟𝑒𝑎𝑠𝑒 𝑖𝑛 𝐴𝑟𝑒𝑎 𝑁𝑒𝑤 𝐴𝑟𝑒𝑎 − 𝑂𝑙𝑑 𝐴𝑟𝑒𝑎 1.69𝑠 2 − 𝑠 2 0.69𝑠 2
= = = = 0.69 = 69%
𝑂𝑙𝑑 𝐴𝑟𝑒𝑎 𝑂𝑙𝑑 𝐴𝑟𝑒𝑎 𝑠2 𝑠2

2.52: Percentage change in side length


If the side length of a square increases by 𝑝%, the area of the square does not depend on the side length of the
square.

Example 2.53
Each side of a square with length 23.75 meters is increased by 30%. The percentage increase in area is:

Since the change does not depend on the side length, take a square of side length 1.

𝑂𝑟𝑖𝑔𝑖𝑛𝑎𝑙 𝑁𝑒𝑤 𝐼𝑛𝑐𝑟𝑒𝑎𝑠𝑒


𝑆𝑖𝑑𝑒 1 1.3
𝐴𝑟𝑒𝑎 1 1.69 0.69

0.69
% 𝑖𝑛𝑐𝑟𝑒𝑎𝑠𝑒 = = 0.69 = 69%
1

Example 2.54
The dimensions of a square are changed so that the length is squared, and the breadth is halved. The old area as
a percentage of the new area is:

2𝑙𝑏 2 200
2
= = %
𝑙 𝑏 𝑙 𝑙

Example 2.55
If the length and width of a rectangle are each increased by 10%, then the perimeter of the rectangle is
increased by what percentage? (AMC 8 1985/19)

P a g e 29 | 83
Get all the files at: https://bit.ly/azizhandouts
Aziz Manva (azizmanva@gmail.com)

Algebraic Method
Let the length and breadth be 𝑙 and 𝑏 respectively
𝑂𝑙𝑑 𝑃𝑒𝑟𝑖𝑚𝑒𝑡𝑒𝑟 = 2(𝑙 + 𝑏)

𝑁𝑒𝑤 𝐿𝑒𝑛𝑔𝑡ℎ = 𝑙 + 10% 𝑜𝑓 𝑙 = 𝑙 + 0.1𝑙 = 1.1𝑙


𝑁𝑒𝑤 𝐵𝑟𝑒𝑎𝑑𝑡ℎ = 𝑏 + 10% 𝑜𝑓 𝑏 = 𝑏 + 0.1𝑏 = 1.1𝑏
𝑁𝑒𝑤 𝑃𝑒𝑟𝑖𝑚𝑒𝑡𝑒𝑟 = 2(1.1𝑙 + 1.1𝑏) = 2.2(𝑙 + 𝑏)

𝑁𝑒𝑤 𝑃 − 𝑂𝑙𝑑 𝑃 2.2(𝑙 + 𝑏) − 2(𝑙 + 𝑏) 0.2(𝒍 + 𝒃) 0.2


% 𝑖𝑛𝑐𝑟𝑒𝑎𝑠𝑒 = = = = = 10%
𝑂𝑙𝑑 𝑃 2(𝑙 + 𝑏) 2(𝒍 + 𝒃) 2

Shortcut Method
Note that in the final calculation, neither the length nor the width appeared. The violet terms above cancelled.
Hence, the answer does not depend on the value of the length, or the value of the width.

Hence, we can take any length and width that we want. For ease of calculations, take
𝐿𝑒𝑛𝑔𝑡ℎ = 𝑊𝑖𝑑𝑡ℎ = 1

𝑂𝑙𝑑 𝑃 = 2(1 + 1) = 2(2) = 4


𝑁𝑒𝑤 𝐿𝑒𝑛𝑔𝑡ℎ = 1 + 10% 𝑜𝑓 1 = 1 + 0.1 = 1.1
𝑁𝑒𝑤 𝐵𝑟𝑒𝑎𝑑𝑡ℎ = 1 + 10% 𝑜𝑓 1 = 1 + 0.1 = 1.1

𝑁𝑒𝑤 𝑃𝑒𝑟𝑖𝑚𝑒𝑡𝑒𝑟 = 2(1.1 + 1.1) = 2(2.2) = 4.4

4.4 − 4 0.4 1
% 𝐼𝑛𝑐𝑟𝑒𝑎𝑠𝑒 = = = 0.1 = = 10%
4 4 10

Example 2.56
If the length of a rectangle is increased by 20% and its width is increased by 50%, then the area is increased by
what percent (AMC 8 1993/21)

Let
𝐿𝑒𝑛𝑔𝑡ℎ = 𝑊𝑖𝑑𝑡ℎ = 100

𝑂𝑙𝑑 𝐴𝑟𝑒𝑎 = 1002 = 10,000


𝑁𝑒𝑤 𝐿𝑒𝑛𝑔𝑡ℎ = 100 + 20% 𝑜𝑓 100 = 100 + 20 = 120
𝑁𝑒𝑤 𝐵𝑟𝑒𝑎𝑑𝑡ℎ = 100 + 50% 𝑜𝑓 100 = 100 + 5 = 150

𝑁𝑒𝑤 𝐴𝑟𝑒𝑎 − 𝑂𝑙𝑑 𝐴𝑟𝑒𝑎 (120)(150) − 10,000 8,000 80


% 𝐼𝑛𝑐𝑟𝑒𝑎𝑠𝑒 = = = = = 80%
𝑂𝑙𝑑 𝐴𝑟𝑒𝑎 10,000 10,000 100

Example 2.57
The length of a rectangle is increased by 10% and the width is decreased by 10%. What percent of the old area
is the new area? (AMC 9 2009/8)

Let 𝐿𝑒𝑛𝑔𝑡ℎ = 𝑊𝑖𝑑𝑡ℎ = 100

𝑂𝑙𝑑 𝐴𝑟𝑒𝑎 = 1002 = 10,000

P a g e 30 | 83
Get all the files at: https://bit.ly/azizhandouts
Aziz Manva (azizmanva@gmail.com)

𝑁𝑒𝑤 𝐿𝑒𝑛𝑔𝑡ℎ = 100 + 10% 𝑜𝑓 100 = 100 + 10 = 110


𝑁𝑒𝑤 𝐵𝑟𝑒𝑎𝑑𝑡ℎ = 100 − 10% 𝑜𝑓 100 = 100 − 10 = 90

𝑁𝑒𝑤 𝐴𝑟𝑒𝑎 (110)(90) 9900 99


= = = = 99%
𝑂𝑙𝑑 𝐴𝑟𝑒𝑎 10,000 10,000 100

F. Maximum and Minimum Area


A rectangle will have maximum area when
𝐿𝑒𝑛𝑔𝑡ℎ = 𝑊𝑖𝑑𝑡ℎ ⇔ 𝑊ℎ𝑒𝑛 𝑖𝑡 𝑖𝑠 𝑎 𝑆𝑞𝑢𝑎𝑟𝑒

If we allow the rectangle to only have integer values for its length and width, then the rectangle with
➢ minimum area will have maximum difference between the length and the width
➢ maximum area will have minimum difference between the length and the width:

Example 2.58
The perimeter of a rectangle with integer sides is 52 units. Find the difference between the maximum and the
minimum possible area of the rectangle.

𝑃 = 2(𝑙 + 𝑤) = 52 ⇒ 𝑙 + 𝑤 = 26

Rectangle with maximum area will have minimum difference between length and width:
𝑙 = 𝑤 = 13 ⇒ 𝐴 = 𝑙𝑤 = 132 = 169

Rectangle with minimum area will have maximum difference between length and width:
𝑙 = 25, 𝑤 = 1 ⇒ 𝐴 = 𝑙𝑤 = 25 × 1 = 25
Difference
169 − 25 = 144

Example 2.59
Rishi is making a rectangle using 50 matchsticks of equal length. He first makes a rectangle with minimum area.
Then, he takes the same matchsticks, and a rectangle with maximum area. Find the difference in the areas of the
two rectangles.

Suppose that each matchstick has length:


1 𝑢𝑛𝑖𝑡

If you are making a rectangle, the perimeter of the rectangle is


𝑃 = 2(𝑙 + 𝑤)

From the condition given in the question, we know that:


2(𝑙 + 𝑤) = 50 ⇒ 𝑙 + 𝑤 = 25

The rectangle with minimum area will have maximum difference between the length and the width:
𝑙 = 24, 𝑤 = 1 ⇒ 𝐴 = 𝑙𝑤 = 24 × 1 = 24

The rectangle with maximum area will have minimum difference between the length and the width:
𝑙 = 13, 𝑤 = 12 ⇒ 𝐴 = 𝑙𝑤 = 13 × 12 = 156

And then, the difference between the two areas is:

P a g e 31 | 83
Get all the files at: https://bit.ly/azizhandouts
Aziz Manva (azizmanva@gmail.com)

156 − 24 = 132

Example 2.60
Ms. Osborne asks each student in her class to draw a rectangle with integer side lengths and a perimeter of 50
units. All of her students calculate the area of the rectangle they draw. What is the difference between the
largest and smallest possible areas of the rectangles?(AMC 8 2008/17)

G. Paths and Borders


Identify a smaller figure inside a larger figure. Then:
𝐴(𝑃𝑎𝑡ℎ) = 𝐴(𝑂𝑢𝑡𝑠𝑖𝑑𝑒 𝐹𝑖𝑔𝑢𝑟𝑒) – 𝐴(𝐼𝑛𝑠𝑖𝑑𝑒 𝐹𝑖𝑔𝑢𝑟𝑒)

Example 2.61: Path Inside


A square garden measuring 20 feet by 30 feet has a three feet marble walkway at its inside border, while the
remaining area is covered by grass. What is the area of the walkway?

𝑂𝑢𝑡𝑒𝑟 𝐴𝑟𝑒𝑎 = 20(30) = 600


𝐼𝑛𝑛𝑒𝑟 𝐴𝑟𝑒𝑎 = (20 − 6)(30 − 6) = 14 × 24 = 336

Area of the path


= [𝑂𝑢𝑡𝑒𝑟] − [𝐼𝑛𝑛𝑒𝑟] = 600 – 336 = 264 𝑠𝑞. 𝑓𝑡

Example 2.62: Path Outside


A rectangular fort(F) measuring 400 meters by 700 meters has a
3-meter moat(M) around it. What is the area of the moat?

A(F + M) – A(C) = 403 * 703 – 400 * 700 = 28000 + 2100 +


1200 + 9 – 28000 = 3309 sq. m.

Example 2.63: Costs


The page of a book which is 30 cm * 40 cm has a margin of 2 cm. The margin is included in the dimensions
given. What percentage of the page is occupied by the margin?

(30*40)-(26*36)=1200-936=264

%=264/1200=22%

Example 2.64: Costs


Find the cost of adding a zari border of 1 inch on each side to a piece of cloth(C) measuring 18 inches by 36
inches, if zari(Z) is available at Rs. 288 per square foot.
A(C + Z) – A(C) = 19 * 37 – 18 * 36 = 703 – 648 = 55 sq. inches
Cost of Zari = 288 * 55/144 = 110 Rs.

If more than one path is drawn through the centre, then the paths will overlap.
𝐴(𝑃𝑎𝑡ℎ) = 𝐴(𝑃𝑎𝑡ℎ − 𝐼) + 𝐴(𝑃𝑎𝑡ℎ − 𝐼𝐼) – 𝐴(𝑂𝑣𝑒𝑟𝑙𝑎𝑝)

P a g e 32 | 83
Get all the files at: https://bit.ly/azizhandouts
Aziz Manva (azizmanva@gmail.com)

Example 2.65
The shaded region formed by the two intersecting perpendicular rectangles, in square
units, is (AMC 8 1988/17)

Method I
The area of the longer rectangle is:
2(10) = 20

The area of the shorter rectangle is:


3(8) = 24

The total area


= 20 + 24 − 2(3) = 38

Method II
2(10) + 3(8 − 2) = 20 + 3(6) = 20 + 18 = 38

Example 2.66
Figure ABCD is a square. Inside this square three smaller squares are drawn with
the side lengths as labeled. The area of the shaded L-shaped region is (AMC 8
2000/6)

We split the 𝐿 − 𝑠ℎ𝑎𝑝𝑒𝑑 region into three rectangles as shown in the diagram.
The total area is:
1(3) + 1(3) + 1(1) = 3 + 3 + 1 = 7

H. Applications

Example 2.67
Charlyn walks completely around the boundary of a square whose sides are each 5 km long. From any point on
her path she can see exactly 1 km horizontally in all directions. What is the area of the region consisting of all
points Charlyn can see during her walk, expressed in square kilometers, and rounded to the nearest whole
number? (AMC 10 2000/18)

Draw the square with sides 5 km each. Note that Charlyn see 1 𝑘𝑚
around her in all directions, which means she can see in the shape of
a circle.

Area of the brown region:


= 𝑂𝑢𝑡𝑒𝑟 𝑆𝑞𝑢𝑎𝑟𝑒 − 𝐼𝑛𝑛𝑒𝑟 𝑆𝑞𝑢𝑎𝑟𝑒
52 − 32 = 25 − 9 = 16

Area of the blue region is four rectangles


= 4(5 × 1) = 20

Area of the green region is four quarter circles that combine to make a complete circle which has area:
= 𝜋𝑟 2 = 𝜋(12 ) = 𝜋 ≈ 3.14

P a g e 33 | 83
Get all the files at: https://bit.ly/azizhandouts
Aziz Manva (azizmanva@gmail.com)

Finally, the total area is


16 + 20 + 3.14 = 39.14 ≈ 39

Example 2.68
A rectangular photograph is placed in a frame that forms a border two inches wide on all sides of the
photograph. The photograph measures 8 inches high and 10 inches wide. What is the area of the border, in
square inches? (AMC 8 2012/6)

The area of the border


= 𝑂𝑢𝑡𝑒𝑟 𝐴𝑟𝑒𝑎 − 𝐼𝑛𝑛𝑒𝑟 𝐴𝑟𝑒𝑎
= 14 ∙ 12 − 10 ∙ 8
= 168 − 80
= 88 𝑖𝑛2

2.69: Arithmetic Progression


An arithmetic progression is a sequence where each successive term increases by
the same amount

Examples of arithmetic progressions are:


5,10,15 ⇒ 𝐼𝑛𝑐𝑟𝑒𝑎𝑠𝑒𝑠 𝑏𝑦 5
3,10,17 ⇒ 𝐼𝑛𝑐𝑟𝑒𝑎𝑠𝑒𝑠 𝑏𝑦 7

Example 2.70
A rug is made with three different colors as shown. The areas of the
three differently colored regions form an arithmetic progression.
The inner rectangle is one foot wide, and each of the two shaded
regions is 1 foot wide on all four sides. What is the length in feet of
the inner rectangle? (AMC 10A 2016/10)

Inner (white) Middle (light gray) Outer (dark


gray)
Width 1 3 5
Length 1 3 5
Area of Rectangle 1 9 25
Area of Colored 1 = 9−1 = 8 = 25 − 9 = 16
Region

1,8,16 𝑑𝑜𝑒𝑠 𝑛𝑜𝑡 𝑓𝑜𝑟𝑚 𝑎𝑛 𝑎𝑟𝑖𝑡ℎ𝑚𝑒𝑡𝑖𝑐 𝑝𝑟𝑜𝑔𝑟𝑒𝑠𝑠𝑖𝑜𝑛

Inner (white) Middle (light gray) Outer (dark gray)


Width 1 3 5
Length 2 4 6
Area of Rectangle 2 12 30
Area of Colored = 12 − 2 = 10 = 30 − 12 = 18
Region

P a g e 34 | 83
Get all the files at: https://bit.ly/azizhandouts
Aziz Manva (azizmanva@gmail.com)

2,10,18 ⇒ 𝐴𝑟𝑖𝑡ℎ𝑚𝑒𝑡𝑖𝑐 𝑃𝑟𝑜𝑔𝑟𝑒𝑠𝑠𝑖𝑜𝑛

Inner (white) Middle (light gray) Outer (dark gray)


Width 1 3 5
Length 𝑥 𝑥+2 𝑥+4
Area of Rectangle 𝑥 3(𝑥 + 2) = 3𝑥 + 6 5(𝑥 + 4) = 5𝑥 + 20
Area of Colored 𝑥 = 2𝑥 + 6 = 2𝑥 + 14
Region

𝑥, 2𝑥 + 6, 2𝑥 + 14

The difference between the second and third terms is:


(2𝑥 + 14) − (2𝑥 + 6) = 8

The difference between the first and second terms is:


(2𝑥 + 6) − 𝑥 = 𝑥 + 6 = 8 ⇒ 𝑥 = 2

I. Composite Figures

Example 2.71
The area of polygon 𝐴𝐵𝐶𝐷𝐸𝐹, in square units, is (AMC 8 1985/4)

5 ∙ 6 + 4 ∙ 5 = 30 + 16 = 46 𝑢𝑛𝑖𝑡𝑠 2

Example 2.72
Three identical rectangles are put together to form rectangle 𝐴𝐵𝐶𝐷 as shown in the
figure below. Given that the length of the shorter side of each of the smaller rectangles
is 5 feet, what is the area in square feet of rectangle 𝐴𝐵𝐶𝐷? (AMC 8 2019/2)

The length of each rectangle is


5 × 2 = 10

The area of each rectangle is


5 × 10 = 50

The total area is


50 × 3 = 150 𝑓𝑡 2

Example 2.73
√3
Four congruent rectangles are placed as shown. If the perimeter of each rectangle is ,
5
then for the outer square, find the
A. perimeter
B. area

P a g e 35 | 83
Get all the files at: https://bit.ly/azizhandouts
Aziz Manva (azizmanva@gmail.com)

√3
𝑃𝑒𝑟𝑖𝑚𝑒𝑡𝑒𝑟 𝑜𝑓 𝑅𝑒𝑐𝑡𝑎𝑛𝑔𝑙𝑒 = 2(𝑙 + 𝑤) =
5
2√3
𝑃𝑒𝑟𝑖𝑚𝑒𝑡𝑒𝑟 𝑜𝑓 𝑂𝑢𝑡𝑒𝑟 𝑆𝑞𝑢𝑎𝑟𝑒 = 4(𝑙 + 𝑤) =
5
3
𝐴𝑟𝑒𝑎 𝑜𝑓 𝑜𝑢𝑡𝑒𝑟 𝑠𝑞𝑢𝑎𝑟𝑒 = (𝑙 + 𝑤)2 =
100

Example 2.74
Four congruent rectangles are placed as shown. The area of the outer square is 4 times that of
the inner square. What is the ratio of the length of the longer side of each rectangle to the
length of its shorter side? (AMC 10A 2009/14)

Method I
Since the question does not give lengths, we assume simple numbers. Let the inner
square have
𝑆𝑖𝑑𝑒 = 1 ⇒ 𝐴𝑟𝑒𝑎 = 1
Then, the outer square has area four times that of the inner square
𝐴𝑟𝑒𝑎(𝑂𝑢𝑡𝑒𝑟) = 4 × 1 = 4 ⇒ 𝑆𝑖𝑑𝑒(𝑂𝑢𝑡𝑒𝑟) = 2

The width of each rectangle


2−1 1
= =
2 2
The length of each rectangle
1 3
=2− =
2 2
The ratio is:
3 1
: = 3: 1
2 2

Method II
(𝑙 + 𝑤)2 = 4(𝑙 − 𝑤)2
𝑙 + 𝑤 = 2(𝑙 − 𝑤)
𝑙 + 𝑤 = 2𝑙 − 2𝑤
𝑙 = 3𝑤
𝑙: 𝑤 = 3: 1

Example 2.75
Four identical squares and one rectangle are placed together to form one large square as
shown. The length of the rectangle is how many times as large as its width? (AMC 10A
2010/2)

Let the side of each smaller square

P a g e 36 | 83
Get all the files at: https://bit.ly/azizhandouts
Aziz Manva (azizmanva@gmail.com)

=1
The side of the larger square
=4
The length of the rectangle
=4
The width of the rectangle
=4−1=3
The length is:
4
𝑡𝑖𝑚𝑒𝑠 𝑜𝑓 𝑡ℎ𝑒 𝑤𝑖𝑑𝑡ℎ
3

J. Inscribed Figures

2.76: Joining the midpoints of a square


The quadrilateral formed by joining the midpoints of a square is itself
a square.

2.77: Area of square formed by joining the midpoints of a square


The area of the square formed by joining the midpoints of the sides of a square is half of the larger square.

Method I
Area of larger square
=2∙2=4

Area of smaller square


= √2 ∙ √2 = 2

Hence, area of smaller square is half of area of larger square.

Method II
Draw a square and connect its midpoints to form another square.
Connect the opposite midpoints to form 4 brown triangles.

There are 4 brown triangles and 4 white triangles.


Each triangle has equal area.

Hence, the area of the brown square


4 1
= = 𝑜𝑓 𝑡ℎ𝑒 𝑙𝑎𝑟𝑔𝑒𝑟 𝑠𝑞𝑢𝑎𝑟𝑒
8 2

P a g e 37 | 83
Get all the files at: https://bit.ly/azizhandouts
Aziz Manva (azizmanva@gmail.com)

Example 2.78
A. Points 𝐴, 𝐵, 𝐶 and 𝐷 are midpoints of the sides of the larger square. If the larger
square has area 60, what is the area of the smaller square? (AMC 8 2006/5)
B. Each of the sides of a square 𝑆1 with area 16 is bisected, and a smaller square 𝑆2 is
constructed using the bisection points as vertices. The same process is carried out
on 𝑆2 to construct an even smaller square 𝑆3 . What is the area of 𝑆3 ? (AMC 10A
2008/10)

Part A
60
𝐴𝑟𝑒𝑎 = = 30
2
Part B
𝑆1 𝑆2 8
𝑆2 = = 8 ⇒ 𝑆3 = = =4
2 2 2

Example 2.79: Geometric Sequence


Square 𝑆 has side length 𝑠. Square 𝑆1 is drawn by connecting the midpoints of 𝑆. Square 𝑆2 is drawn by
connecting the midpoint of 𝑆1 . The process continues. What is the area of 𝑆10? 𝑆𝑛

𝐴𝑟𝑒𝑎(𝑆) = 𝑠 2
𝑠2
𝑆1 =
2
𝑠2
𝑆1 𝑠2 1 𝑠2 𝑠2
𝑆2 = = 2 = × = = 2
2 2 2 2 4 2
𝑠2
𝑆2 2 𝑠2
𝑆3 = = 2 = 3
2 2 2
𝑠2
𝑆10 = 10
2
𝑠2
𝑆𝑛 = 𝑛
2

Example 2.80
In rectangle 𝐴𝐵𝐶𝐷, 𝐴𝐵 = 6 and 𝐴𝐷 = 8. Point 𝑀 is the midpoint of 𝐴𝐷. What is the area of △ 𝐴𝑀𝐶? (AMC 8
2016/2)

The area of the triangle is


1 1 1
ℎ𝑏 = (𝐴𝑀)(𝐷𝐶) = (4)(6) = 12
2 2 2

P a g e 38 | 83
Get all the files at: https://bit.ly/azizhandouts
Aziz Manva (azizmanva@gmail.com)

K. Trisection

Example 2.81
Each side of the large square in the figure is trisected (divided into three equal parts).
The corners of an inscribed square are at these trisection points, as shown. The ratio of
the area of the inscribed square to the area of the large square is (AMC 8 1997/15)

Method I
Area of each triangle
1 1
= ℎ𝑏 = (1)(2) = 1
2 2
Using complementary areas, area of the smaller square is
𝐴(𝑆𝑚𝑎𝑙𝑙𝑒𝑟 𝑆𝑞𝑢𝑎𝑟𝑒) = 𝐴(𝐿𝑎𝑟𝑔𝑒𝑟 𝑆𝑞𝑢𝑎𝑟𝑒) − 𝐴(𝑇𝑟𝑖𝑎𝑛𝑔𝑙𝑒𝑠)
= 32 − 1(4) = 9 − 4 = 5
The ratio is
5
9
Method II
By Pythagoras Theorem, the side length of the smaller square is:
= √12 + 22 = √5
Area of the larger square is
2
(√5) = 5
The ratio is
5 5
2
=
3 9
L. Complementary Areas

Example 2.82
The area of rectangle 𝐴𝐵𝐶𝐷 is 72. If point 𝐴 and the midpoints of 𝐵𝐶 and 𝐶𝐷 are joined
to form a triangle, the area of that triangle is (AMC 8 2000/25)

We can take numbers and do this. The area of the triangles is the same irrespective of the numbers:

The area is
72 − (18 + 18 + 9) = 72 − 45 = 27

Example 2.83
In square 𝐴𝐵𝐶𝐸, 𝐴𝐹 = 2𝐹𝐸 and 𝐶𝐷 = 2𝐷𝐸. What is the ratio of the area of Δ𝐵𝐹𝐷 to
the area of square 𝐴𝐵𝐶𝐸? (AMC 8 2008/23)

P a g e 39 | 83
Get all the files at: https://bit.ly/azizhandouts
Aziz Manva (azizmanva@gmail.com)

9 − 6.5 2.5 5
= = = 5: 18
9 9 18

Example 2.84
A square with area 4 is inscribed in a square with area 5, with one vertex of the smaller
square on each side of the larger square. A vertex of the smaller square divides a side of
the larger square into two segments, one of length 𝑎, and the other of length 𝑏. What is
the value of 𝑎𝑏? (AMC 8 2012/25)

The area of the four white triangles is:


5−4 = 1
Area of each triangle
𝑎𝑏
=
2
Area of four triangles
𝑎𝑏
= 4 ( ) = 2𝑎𝑏
2

1
2𝑎𝑏 = 1 ⇒ 𝑎𝑏 =
2

Example 2.85
One-inch squares are cut from the corners of this 5 inch square. What is the area in square
inches of the largest square that can fit into the remaining space? (AMC 8 2015/25)

The area of the inner square


= 32 = 9

The areas of the four triangles


1 1
= 4 ( ℎ𝑏) = 4 ( ∙ 1 ∙ 3) = 6
2 2

The total area is


= 9 + 6 = 15

P a g e 40 | 83
Get all the files at: https://bit.ly/azizhandouts
Aziz Manva (azizmanva@gmail.com)

Example 2.86
In the figure, the length of side 𝐴𝐵 of square 𝐴𝐵𝐶𝐷 is √50 and 𝐵𝐸 = 1. What is the
area of the inner square 𝐸𝐹𝐺𝐻? (AMC 10A 2005/8)

Step I: Prove triangles congruent


∠𝐴𝐻𝐵 = ∠𝐴𝐺𝐷 = 90°
Let
∠𝐴𝐵𝐻 = 𝛼 ⇒ ∠𝐵𝐴𝐻 = 90 − 𝛼

Since ∠𝐵𝐴𝐷 = 90°:


∠𝐵𝐴𝐻 + ∠𝐺𝐴𝐷 = 90°
∠𝐺𝐴𝐷 = 90 − ∠𝐵𝐴𝐻 = 90 − (90 − 𝛼) = 𝛼

By complementary angles in a right triangle:


∠𝐺𝐷𝐴 = 90 − ∠𝐺𝐴𝐷 = 90 − 𝛼

From the diagram, Δ𝐵𝐴𝐻 ≅ ΔDAG by 𝐴𝑆𝐴 𝐶𝑜𝑛𝑔𝑟𝑢𝑒𝑛𝑐𝑒:


𝐵𝐴 = 𝐷𝐴 = √50
∠𝐵𝐴𝐻 = ∠𝐺𝐷𝐴 = 90 − 𝛼
∠𝐴𝐵𝐻 = ∠𝐺𝐴𝐷 = 𝛼
By 𝐶𝑃𝐶𝑇𝐶:
𝐴𝐺 = 𝐵𝐻
𝐺𝐻 + 𝐻𝐴 = 𝐻𝐸 + 𝐸𝐵
𝐻𝐴 = 𝐸𝐵 = 1

Step II: Use the Congruence

∠𝐴𝐻𝐵 = 180 − 90 = 90°

By Pythagoras Theorem in Δ𝐴𝐻𝐵


𝐴𝐻 2 + 𝐻𝐵2 = 𝐴𝐵2

Substitute 𝐴𝐻 = 1, 𝐻𝐵 = 𝐻𝐸 + 𝐸𝐵 = 𝐻𝐸 + 1, 𝐴𝐵 = √50
2
12 + (𝐻𝐸 + 1)2 = (√50)
1 + (𝐻𝐸 + 1)2 = 50
(𝐻𝐸 + 1)2 = 49

Take the square root both sides:


𝐻𝐸 + 1 = 7
𝐻𝐸 = 6
𝐻𝐸 2 = 36

P a g e 41 | 83
Get all the files at: https://bit.ly/azizhandouts
Aziz Manva (azizmanva@gmail.com)

Example 2.87
In rectangle 𝐴𝐷𝐸𝐻, points 𝐵 and 𝐶 trisect 𝐴𝐷, and points 𝐺 and 𝐹 trisect 𝐻𝐸. In addition, 𝐴𝐻 = 𝐴𝐶 = 2, and
𝐴𝐷 = 3. What is the area of quadrilateral 𝑊𝑋𝑌𝑍 shown in the figure? (AMC 10A 2006/17)

M. Ratios

Example 2.88
A square is drawn inside a rectangle. The ratio of the width of the rectangle to a side of the square is 2:1. The
ratio of the rectangle's length to its width is 2:1. What percent of the rectangle's area is in the square?
(AMC 10A 2008/2)

Since the question uses ratios, we can take any side lengths that
we want.

Take a square with side length 1. The rectangle will have width
1×2=2

And it will have length


2×2=4

The area of the rectangle will be


2×4=8

And the percentage will be


1
= 12.5%
8

Example 2.89
Older television screens have an aspect ratio of 4: 3. That is, the ratio of the width to the
height is 4: 3. The aspect ratio of many movies is not 4: 3, so they are sometimes shown
on a television screen by "letterboxing" - darkening strips of equal height at the top and
bottom of the screen, as shown. Suppose a movie has an aspect ratio of 2:1 and is shown
on an older television screen with a 27-inch diagonal. What is the height, in inches, of

P a g e 42 | 83
Get all the files at: https://bit.ly/azizhandouts
Aziz Manva (azizmanva@gmail.com)

each darkened strip? (AMC 10A 2008/14)

The TV has height and width in the ratio 4: 3. The movie has height which is
half of its width.
4
𝐻𝑒𝑖𝑔ℎ𝑡 𝑜𝑓 𝑚𝑜𝑣𝑖𝑒 = = 2
2
The height of the strip remaining has:
4−2 2
𝐻𝑒𝑖𝑔ℎ𝑡 = = =1
2 2
This gives us the diagram to the right, which has diagonal
(3,4,5) ⇒ 𝐷𝑖𝑎𝑔𝑜𝑛𝑎𝑙 = 5

Double the dimensions to get a diagonal of 10, and then multiply by 2.7 to get a diagonal of 27.

Method II
27
Scale the height of the side length by to get the height in the TV screen in the question:
5
1 1 27
× × 27 = = 2.7 𝑖𝑛𝑐ℎ𝑒𝑠
2 5 10

Example 2.90
A paint brush is swept along both diagonals of a square to produce the symmetric painted area, as shown. Half
the area of the square is painted. What is the ratio of the side length of the square to the brush width? (AMC 10A
2007/19)

Example 2.91
In the question below, mark all options that are correct.
If two adjacent sides and a diagonal of a quadrilateral form a right-angled triangle, then that quadrilateral is
definitely a:
A. Rhombus
B. Square
C. Rectangle
D. Parallelogram
E. Trapezium
F. None of these

P a g e 43 | 83
Get all the files at: https://bit.ly/azizhandouts
Aziz Manva (azizmanva@gmail.com)

The quadrilateral in the diagram does not fall under Options A to E, though A
it meets the conditions in the question. B
Hence, Option F. C
D
N. Triangles in a Rectangle

Example 2.92
The diagram alongside shows a rectangle with dimension 3 × 4. If the
triangle on the left has area which is one third of the triangle on the right,
then determine the area of the smaller triangle.

Area of the Triangles


1 1 3
𝐴𝑟𝑒𝑎 𝑜𝑓 𝐼 = ℎ𝑏 = (3)𝑏1 = 𝑏1
2 2 2
1 1 3
𝐴𝑟𝑒𝑎 𝑜𝑓 𝐼𝐼 = ℎ𝑏 = (3)𝑏2 = 𝑏2
2 2 2
The total area is
3 3 3 3
𝑏1 + 𝑏2 = (𝑏1 + 𝑏2 ) = (4) = 6
2 2 2 2

Dividing the area


The ratio of the area of the triangle is:
1: 3 ⇒ 1 + 3 = 4
The smaller triangle has area
1 1
𝑇𝑜𝑡𝑎𝑙 𝑎𝑟𝑒𝑎 = 𝑜𝑓 6 = × 6 = 1.5
4 4

2.2 Visual Techniques


A. Visual Techniques

2.93: Rearrangement
In certain cases, a polygon can be rearranged without changing its perimeter.

Example 2.94
The perimeter of the polygon shown (i n the given units) is (AMC 8 1986/13)

Consider a rectangle that has the same perimeter as the given polygon.
(Make sure you understand why the perimeters are the same.)

Hence, the perimeter of the polygon


= 𝑃(𝑅𝑒𝑐𝑡𝑎𝑛𝑔𝑙𝑒) = 2(6 + 8) = 2 × 14 = 28

P a g e 44 | 83
Get all the files at: https://bit.ly/azizhandouts
Aziz Manva (azizmanva@gmail.com)

2.95: Adding a Tile


If you have a tiled shape made of squares with side length 𝑠, and you add a square to the shape, you will
➢ NOT change the perimeter if the new square shares two sides with the old shape
➢ Increase the perimeter by 2𝑠 if the new square shares a single side with the old shape

Example 2.96
𝑀𝑎𝑟𝑘 𝑎𝑙𝑙 𝑐𝑜𝑟𝑟𝑒𝑐𝑡 𝑜𝑝𝑡𝑖𝑜𝑛𝑠
Eight 1 × 1 square tiles are arranged as shown so their outside
edges form a polygon with a perimeter of 14 units. Two
additional tiles of the same size are added to the figure so that
at least one side of each tile is shared with a side of one of the
squares in the original figure. Find all possible values of the
perimeter of the new figure? (AMC 8 1992/22, Adapted)

Case I: Perimeter 14
If you add a tile at the top left and the bottom right, the perimeter of the
new figure remains the same, since the blue dotted lines get added, and
the purple lines get removed.

Case II: Perimeter 16


If you add a tile at the top left and top right, the perimeter
increases by 2.

Case III: Perimeter 18


If you add two tiles at the right, the perimeter increases
by 4.

Example 2.97
Suppose a square piece of paper is folded in half vertically. The folded paper is then cut in
half along the dashed line. Three rectangles are formed-a large one and two small ones.
What is the ratio of the perimeter of one of the small rectangles to the perimeter of the large
rectangle? (AMC 8 1989/24)

P a g e 45 | 83
Get all the files at: https://bit.ly/azizhandouts
Aziz Manva (azizmanva@gmail.com)

Perimeter of small rectangle


= 2(1 + 4) = 2(5)

Perimeter of large rectangle


= 2(2 + 4) = 2(6)

Ratio of perimeter of small rectangle to large rectangle


= 2(5): 2(6) = 5: 6

Example 2.98
A square is divided, as shown. What fraction of the area of the square is shaded? (Gauss
Grade 7 2007/23)

Method I
Divide the square into four equal parts using its diagonals. See 1st diagram on the left. The
light brown triangle has area
1
𝑜𝑓 𝑡ℎ𝑒 𝑠𝑞𝑢𝑎𝑟𝑒
4

Divide the light brown triangle into two


equal parts, getting a dark brown triangle,
which has area
1 1 1
× =
2 4 8

Add a further dark brown triangle to get the


shape that we want. The smaller dark brown triangle has area
1 1 1
× =
2 8 16

The total area is then:


1 1 3
+ =
8 16 16

Method II:
Divide the square into 16 triangles of equal area as
shown. The shaded area occupies 3 of the 16
triangles, and hence the required ratio is
3
16

P a g e 46 | 83
Get all the files at: https://bit.ly/azizhandouts
Aziz Manva (azizmanva@gmail.com)

Example 2.99
In the diagram shown:
𝑆𝑇𝑈𝑉 is a square
𝑄 and 𝑃 are the midpoints of 𝑆𝑇 and 𝑈𝑉
𝑃𝑅 = 𝑄𝑅, and
𝑉𝑄 is parallel to 𝑃𝑅
What is the ratio of the shaded area to the unshaded area? (Gauss Grade 7 2014/24)

Let the total area of 𝑆𝑇𝑈𝑉 be 1.


1
𝐴(Δ𝑄𝑉𝑃) =
4
Also,
𝑃𝑅 ∥ 𝑉𝑄 ⇒ 𝑃𝑅𝑇 𝑖𝑠 𝑎 𝑑𝑖𝑎𝑔𝑜𝑛𝑎𝑙 𝑜𝑓 𝑄𝑇𝑈𝑃
𝑄𝑅 = 𝑃𝑅 ⇒ ∠𝑃𝑄𝑈 = ∠𝑄𝑃𝑇 ⇒ 𝑄𝑅𝑈 𝑖𝑠 𝑎 𝑑𝑖𝑎𝑔𝑜𝑛𝑎𝑙 𝑜𝑓 𝑄𝑇𝑈𝑃

Since diagonals of a rectangle divide it into four triangles with equal area:
1 1 1 1
𝐴(𝑄𝑅𝑃) = 𝐴(𝑄𝑇𝑈𝑃) = ∙ =
4 4 2 8
Total Shaded Area
1 1 3
= + =
4 8 8
Total Unshaded Area
3 5
=1− =
8 8
Ratio of Unshaded to Shaded Area
3 5
= : = 3: 5
8 8

Example 2.100
Dots are spaced one unit apart, horizontally and vertically. The number of square units
enclosed by the polygon is (AMC 8 1998/6)

Move the top triangle and fit in the bottom space, giving a rectangle with area
2×3=6

Example 2.101
What fraction of the large 12 by 18 rectangular region is shaded? (AMC 8
1987/12)

We can calculate the area of the region.

P a g e 47 | 83
Get all the files at: https://bit.ly/azizhandouts
Aziz Manva (azizmanva@gmail.com)

Let the given shaded region have area


1

The bottom right rectangle will have area


3

The bottom left rectangle will also have area


3

And the top rectangle will have area


6

Total area
= 3 + 3 + 6 = 12

And the required fraction will be:


1
12

Example 2.102
What fraction of the square is shaded? (AMC 8 1990/3)

Reflect two of the grey shaded parts across the diagonal of the square.
Note the entire area to the right of the diagonal is now shaded.

By symmetry, the two areas are equal. The fraction is


1
2

Example 2.103

P a g e 48 | 83
Get all the files at: https://bit.ly/azizhandouts
Aziz Manva (azizmanva@gmail.com)

Determine the number of unshaded squares in the diagram alongside. All of the smallest squares have side
length 1.

Count the squares as shown to the right. We get:


1,3,5,7,9
As the number of squares in each line.

And the number of unshaded squares is:


1 + 5 + 9 = 15

Example 2.104
What fraction of this square region is shaded? Stripes are equal in width, and the figure is
drawn to scale. (AMC 8 1997/10)

Use the square at the bottom left as a unit square, and see the pattern:
1,3,5,7, 9,11
Unshaded area:
1 + 5 + 9 15 5
= = =
36 36 12
Shaded Area
5 7
=1− =
12 12

Example 2.105
Suppose the figure at right is extended using the same pattern, but so that it is a square
with a side length of 20. Find the shaded area.

1,3,5, … ,39
Where
1 𝑖𝑠 𝑡ℎ𝑒 1𝑠𝑡 𝑜𝑑𝑑 𝑛𝑢𝑚𝑏𝑒𝑟 = 2(1) − 1 = 1
3 𝑖𝑠 𝑡ℎ𝑒 2𝑛𝑑 𝑜𝑑𝑑 𝑛𝑢𝑚𝑏𝑒𝑟 = 2(2) − 1 = 4 − 1 = 3
39 𝑖𝑠 𝑡ℎ𝑒 20𝑡ℎ 𝑜𝑑𝑑 𝑛𝑢𝑚𝑏𝑒𝑟 = 2(20) − 1 = 40 − 1 = 39

Out of these, we want the sum of alternate numbers:


3 + 7 + 11 + 15 + 19 + 23 + 27 + 31 + 35 + 39

P a g e 49 | 83
Get all the files at: https://bit.ly/azizhandouts
Aziz Manva (azizmanva@gmail.com)

Arrange the numbers in pairs:


= (3 + 39) + (7 + 35) + (11 + 31) + (15 + 27) + (19 + 23)
= 42 + 42 + 42 + 42 + 42
= 5 × 42
= 10 × 21
= 210

Example 2.106
What is the ratio of the area of the shaded square to the area of the large square?
(The figure is drawn to scale) (AMC 8 1998/13)

Divide the square into four equal parts by drawing the


blue diagonals.

Further divide the bottom diagonal into four congruent


brown triangles.

The required area is then:


2 1
=
16 8

Example 2.107
Let 𝑃𝑄𝑅𝑆 be a square piece of paper. 𝑃 is folded onto 𝑅 and then 𝑄 is folded onto 𝑆. The
area of the resulting figure is 9 square inches. Find the perimeter of square 𝑃𝑄𝑅𝑆. (AMC 8
1998/20)

Fold the square in half to get Δ𝑄𝑆𝑅. The makes the


area half of what is was.
Fold Δ𝑄𝑆𝑅 in half to get a smaller triangle. This makes
1𝑡ℎ
the area 4 of the original square.

Area of square 𝑃𝑄𝑅𝑆


= 4(𝐴𝑟𝑒𝑎 𝑜𝑓 𝑟𝑒𝑠𝑢𝑙𝑡𝑖𝑛𝑔 𝑓𝑖𝑔𝑢𝑟𝑒) = 4(9) = 36

Side length of 𝑃𝑄𝑅𝑆


= √36 = 6

Perimeter of 𝑃𝑄𝑅𝑆
= 4 ∙ 6 = 24

Example 2.108
A corner of a tiled floor is shown. If the entire floor is tiled in this way and
each of the four corners looks like this one, then what fraction of the tiled
floor is made of darker tiles? (AMC 8 2002/23)

P a g e 50 | 83
Get all the files at: https://bit.ly/azizhandouts
Aziz Manva (azizmanva@gmail.com)

The pattern repeats every six tiles. See the


blue square created in the left diagram.
Further, the blue square can be divided into
four brown squares, each of which has equal
shading.

Then the shading in the brown square

The required shading is


1 1
+1+1+1+ =4
2 2
The total number of squares
3×3=9

The fraction is
4
9

Example 2.109
In the figure, what is the ratio of the area of the gray squares to the area of the white
squares? (AMC 8 2008/6)

6: 10 = 3: 5

Example 2.110
Each of the following four large congruent squares is subdivided into
combinations of congruent triangles or rectangles and is partially bolded.
What percent of the total area is partially bolded? (AMC 8 2011/7)

Visually rearrange the shapes.


Bottom right occupies a rectangle.
Top left occupies a rectangle.
Top right occupies a triangle at the top left.

P a g e 51 | 83
Get all the files at: https://bit.ly/azizhandouts
Aziz Manva (azizmanva@gmail.com)

Overall, we get
1
= 25%
4

Example 2.111
The diagram represents a 7-foot-by-7-foot floor that is tiled with 1-square-foot black
tiles and white tiles. Notice that the corners have white tiles. If a 15-foot-by-15-foot
floor is to be tiled in the same manner, how many white tiles will be needed? (AMC 9
2009/18)

Size of floor
1 𝐹𝑜𝑜𝑡: 1 𝑊ℎ𝑖𝑡𝑒 𝑇𝑖𝑙𝑒
3 𝐹𝑜𝑜𝑡: 4 = 22 𝑊ℎ𝑖𝑡𝑒 𝑇𝑖𝑙𝑒𝑠
5 𝐹𝑜𝑜𝑡: 9 = 32 𝑊ℎ𝑖𝑡𝑒 𝑇𝑖𝑙𝑒𝑠
7 𝐹𝑜𝑜𝑡: 42 𝑊ℎ𝑖𝑡𝑒 𝑇𝑖𝑙𝑒𝑠
9 𝐹𝑜𝑜𝑡: 52 𝑊ℎ𝑖𝑡𝑒 𝑇𝑖𝑙𝑒𝑠
11 𝐹𝑜𝑜𝑡: 62 𝑊ℎ𝑖𝑡𝑒 𝑇𝑖𝑙𝑒𝑠
13 𝐹𝑜𝑜𝑡: 72 𝑊ℎ𝑖𝑡𝑒 𝑇𝑖𝑙𝑒𝑠
15 𝐹𝑜𝑜𝑡: 82 𝑊ℎ𝑖𝑡𝑒 𝑇𝑖𝑙𝑒𝑠

Example 2.112
Extend the square pattern of 8 black and 17 white square tiles by attaching a border
of black tiles around the square. What is the ratio of black tiles to white tiles in the
extended pattern? (AMC 8 2011/3)

The number of black tiles which get added is:


5 × 4 + 4 = 20 + 4 = 24

The total black tiles are now:


8 + 24 = 32

The ratio is:


32: 17

P a g e 52 | 83
Get all the files at: https://bit.ly/azizhandouts
Aziz Manva (azizmanva@gmail.com)

Example 2.113
Each of the three large squares shown below is
the same size. Segments that intersect the sides
of the squares intersect at the midpoints of the
sides. How do the shaded areas of these squares
compare? (AMC 8 1994/12)
A. The shaded areas in all three are equal.
B. Only the shaded areas of I and II are equal.
C. Only the shaded areas of I and III are equal.
D. Only the shaded areas of II and III are equal.
E. The shaded areas of I, II and III are all different.

Example 2.114
The perimeter of one square is 3 times the perimeter of another square. The area of the larger square is how
many times the area of the smaller square? (AMC 8 1994/16)

Let
𝑝𝑆 = 4 ⇒ 𝑆𝑖𝑑𝑒𝑆 = 1 ⇒ 𝐴𝑟𝑒𝑎 = 1
12
𝑝𝐿 = 12 ⇒ 𝑆𝑖𝑑𝑒𝐿 = = 3 ⇒ 𝐴𝑟𝑒𝑎 = 9
4

9
𝑅𝑎𝑡𝑖𝑜 𝑜𝑓 𝑎𝑟𝑒𝑎𝑠 = ⇒ 9 𝑡𝑖𝑚𝑒𝑠
1

Example 2.115
The area of each of the four congruent L-shaped regions of this 100-inch by 100-inch
3
square is of the total area. How many inches long is the side of the center square?
16
(AMC 8 1995/18)

P a g e 53 | 83
Get all the files at: https://bit.ly/azizhandouts
Aziz Manva (azizmanva@gmail.com)

The total area of the 𝐿 − 𝑠ℎ𝑎𝑝𝑒𝑑 regions is:


3 3
×4=
16 4

The area of the center square


3 1
=1− = 𝑜𝑓 𝑡ℎ𝑒 𝑒𝑛𝑡𝑖𝑟𝑒 𝑠𝑞𝑢𝑎𝑟𝑒
4 4

The actual area


1 10,000
= (1002 ) = = 2500
4 4

The side length is:


√2500 = √25 × √100 = 5 × 10 = 50

Example 2.116
Two congruent squares, 𝐴𝐵𝐶𝐷 and 𝑃𝑄𝑅𝑆, have side length 15. They overlap to
form the 15 by 25 rectangle 𝐴𝑄𝑅𝐷 shown. What percent of the area of rectangle
𝐴𝑄𝑅𝐷 is shaded? (AMC 8 2011/13)

The length of the shaded portion


= 15 + 15 − 25 = 30 − 25 = 5

The percent of area that is shaded is in proportion to the length that is


shaded because the height is the same throughout:
5
= = 20%
25
Method II: Algebra
𝑥 + 2(15 − 𝑥) = 25
𝑥 + 30 − 2𝑥 = 25
𝑥=5

Example 2.117
Two squares 17 𝑐𝑚 × 17 𝑐𝑚 overlap to form a rectangle 17 𝑐𝑚 × 30 𝑐𝑚. The area of the overlapping region is:
(NMTC Sub-Junior/Screening, 2011/3)

If you do not overlap the squares, and put them next to each you get:

If you overlap and move the right square to the left, the length will reduce.
To reduce the length by 34 − 30 = 4 𝑐𝑚, you move the right square 4𝑐𝑚 to the left.

P a g e 54 | 83
Get all the files at: https://bit.ly/azizhandouts
Aziz Manva (azizmanva@gmail.com)

The area of this overlap is


4 × 17 = 68 𝑐𝑚2

B. Simple Cutting

Example 2.118
A square with side length 8 is cut in half, creating two congruent rectangles. What are the dimensions of one of
these rectangles? (AMC 10A 2012/2)

The dimensions are:



4 × ⏟
8
𝑩𝒓𝒆𝒂𝒅𝒕𝒉 𝑳𝒆𝒏𝒈𝒕𝒉

2.119: Types of Angles


If you cut rectangular, non-overlapping regions from a rectangle, the
perimeter does not change.

➢ Does not “change” can be written as does not “vary”, and a single
word for this is 𝑖𝑛𝑣𝑎𝑟𝑖𝑎𝑛𝑡.
➢ If the cuts overlap, then the perimeter does not remain the same.

Example 2.120
A. Tanmay bought a rectangular carpet. He cut a rectangle from the top left corner of the carpet. He cut
another rectangle from the top right corner of the carpet. The cuts did not overlap. The carpet originally
had dimensions 3 × 4. What was the perimeter of the carpet after the cuts were made.
B. If the cuts that Tanmay made had been overlapping, what would have been your answer to Part A.

Part A
The original carpet had
𝑃𝑒𝑟𝑖𝑚𝑒𝑡𝑒𝑟 = 2(3 + 4) = 2(7) = 14

The new carpet (after the cuts) will have the same perimeter
= 14
Part B
Cannot be determined without further information since the cuts are overlapping.

Example 2.121
Tulsi bought a square-shaped carpet. She removed two unequal, nonoverlapping rectangular regions from two
of its corners. Now she could fit the carpet exactly into her study room. If the perimeter of the study room is
16 𝑚, what is the area of the original carpet? (NMTC Primary-Screening, 2008/16)

P a g e 55 | 83
Get all the files at: https://bit.ly/azizhandouts
Aziz Manva (azizmanva@gmail.com)

The perimeter of the square


= 𝑃𝑒𝑟𝑖𝑚𝑒𝑡𝑒𝑟 𝑜𝑓 𝑆𝑡𝑢𝑑𝑦 𝑅𝑜𝑜𝑚 = 16𝑚

The side length of the square:


𝑃𝑒𝑟𝑖𝑚𝑒𝑡𝑒𝑟 16
= = = 4𝑚
4 4
The area of the square
= 𝑠 2 = 42 = 16𝑚2

Example 2.122
A square with integer side length is cut into 10 squares, all of which have integer side length and at least 8 of
which have area 1. What is the smallest possible value of the length of the side of the original square? (AMC 8
2012/17)

There are 10 squares. Each square has integer side length. Hence, area of these 10
squares must be minimum:
10 × 1 = 10

A square of side length will have area


3 × 3 = 9 < 10 ⇒ 𝐷𝑜𝑒𝑠 𝑛𝑜𝑡 𝑤𝑜𝑟𝑘

Let us try a square of side length 4. We make 8 squares of area 1, and 2 squares of
area 4, and we are able to cover the whole square.

Hence,
𝑆𝑚𝑎𝑙𝑙𝑒𝑠𝑡 𝑝𝑜𝑠𝑠𝑖𝑏𝑙𝑒 𝑣𝑎𝑙𝑢𝑒 = 4

P a g e 56 | 83
Get all the files at: https://bit.ly/azizhandouts
Aziz Manva (azizmanva@gmail.com)

3. QUADRILATERALS
3.1 Sum of Angles
Example 3.1
In the figure, ∠𝐴, ∠𝐵, and ∠𝐶 are right angles. If ∠𝐴𝐸𝐵 = 40∘ and
∠𝐵𝐸𝐷 = ∠𝐵𝐷𝐸, then ∠𝐶𝐷𝐸 = (AMC 8 1995/13)

In Isosceles Right Δ𝐵𝐸𝐷


∠𝐵𝐸𝐷 = ∠𝐵𝐷𝐸 = 45°

Sum of angles of a Quadrilateral is 360°. Hence:


∠𝐶𝐷𝐸 = 360 − 90°
⏟ − 90°
⏟ − 40°
⏟ − 40°

∠𝐴 ∠𝐶 ∠𝐴𝐸𝐵 ∠𝐵𝐸𝐷

Example 3.2
The angles of quadrilateral 𝐴𝐵𝐶𝐷 satisfy ∠𝐴 = 2 ∠𝐵 = 3 ∠𝐶 = 4 ∠𝐷. What is the degree measure of ∠𝐴,
rounded to the nearest whole number? (AMC 10B 2007/15)

𝑎 = 2𝑏 = 3𝑐 = 4𝑑 = 𝑘
𝑘 𝑘 𝑘 360 × 12
𝑘 + + + = 360 ⇒ 𝑘 = = 172.8 ≈ 173
2 3 4 25

3.3: Area Formula


Consider quadrilateral 𝐴𝐵𝐶𝐷 with diagonal 𝐵𝐷. Also
𝐴𝑋 ⊥ 𝐵𝐷, 𝐶𝑌 ⊥ 𝐵𝐷

Then, the area of quadrilateral 𝐴𝐵𝐶𝐷 is:


1
(𝐵𝐷)(𝐴𝑋 + 𝐶𝑌)
2

1 1
𝐴(Δ𝐴𝐵𝐷) = ℎ𝑏 = (𝐵𝐷)(𝐴𝑋)
2 2
1 1
𝐴(ΔC𝐵𝐷) = ℎ𝑏 = (𝐵𝐷)(𝐶𝑌)
2 2

1 1 1
𝐴(𝐴𝐵𝐶𝐷) = [𝐴𝐵𝐷] + [𝐶𝐵𝐷] = (𝐵𝐷)(𝐴𝑋) + (𝐵𝐷)(𝐶𝑌) = (𝐵𝐷)(𝐴𝑋 + 𝐶𝑌)
2 2 2

Example 3.4
In quadrilateral ABCD, with diagonal BD, 𝐴𝑋 and 𝐶𝑌 are perpendicular to 𝐵𝐷.
Find the area of the quadrilateral if:
A. 𝐵𝐷 = 13, 𝐴𝑋 = 5, 𝐶𝑌 = 4
B. 𝐵𝐷 = 12, 𝐴𝑋 = 4, 𝐶𝑌 = 7

Part A
1 1
𝐴(𝐴𝐵𝐶𝐷) = (𝐵𝐷)(𝐴𝑋 + 𝐶𝑌) = (12)(11) = 66
2 2
Part B

P a g e 57 | 83
Get all the files at: https://bit.ly/azizhandouts
Aziz Manva (azizmanva@gmail.com)

1 1 117
𝐴(𝐴𝐵𝐶𝐷) = (𝐵𝐷)(𝐴𝑋 + 𝐶𝑌) = (13)(5 + 4) =
2 2 2

Example 3.5
The perimeter of a parallelogram is 30 𝑐𝑚. If one of the sides measures 5 𝑐𝑚, find the length of the other three
sides.

Example 3.6
The adjacent sides of a parallelogram are in the ratio 2: 5. If the perimeter of the parallelogram is 10, find the
sides of the parallelogram.

2𝑥 + 5𝑥 + 2𝑥 + 5𝑥 = 10
14𝑥 = 10
10 5
𝑥= =
14 7

10 25
2𝑥 = , 5𝑥 =
7 7
Example 3.7
A piece of wire 1 m long is used to make four squares of equal size. Each of the squares is then opened up and
cut into four smaller, equal, squares. Find the side length of the smaller squares in 𝑚.

In all, we are making


4 ×= 16 𝑠𝑞𝑢𝑎𝑟𝑒𝑠
Total length of wire
1𝑚
Each square has
1 1 1 1 1 1
𝑃𝑒𝑟𝑖𝑚𝑒𝑡𝑒𝑟 = 1 × × = 𝑚 ⇒ 𝑆𝑖𝑑𝑒 = × = 𝑚
4 4 16 16 4 64

Example 3.8
A rectangular garden of dimensions 15𝑚 × 25𝑚 has to have four
flowerbeds added to its corners. The garden is currently all grass. Under
plan
➢ A: The flowerbeds will be in the shape of rectangles of dimension
2𝑚 × 2𝑚, planted with 𝑔𝑒𝑟𝑚𝑎𝑛𝑖𝑢𝑚𝑠.
➢ B: The flowerbeds will be in the shape of isosceles right-angled
triangles with length of leg 3𝑚, planted with 𝑏𝑜𝑢𝑔𝑒𝑛𝑣𝑖𝑙𝑙𝑒.
Find under each plan:
A. the area of the remaining grass.
B. the cost of the flowers

Area of garden
= 15 × 25 = 375 𝑚2
Area of rectangular flowerbeds
= 4 × 2 × 2 = 16 𝑚2
Area of triangular flowerbeds
1 1
= 4 × ℎ𝑏 = 4 × × 3 × 3 = 18 𝑚2
2 2

P a g e 58 | 83
Get all the files at: https://bit.ly/azizhandouts
Aziz Manva (azizmanva@gmail.com)

Part A
𝑃𝑙𝑎𝑛 𝐴 = 375 − 16 = 359 𝑚2
𝑃𝑙𝑎𝑛 𝐵 = 375 − 18 = 357 𝑚2
Part B
𝑃𝑙𝑎𝑛 𝐴 =

Example 3.9
In a rectangular park, the grass (green area) is surrounded by a walking
track (2 𝑚 𝑤𝑖𝑑𝑡ℎ, dark shaded area), which is surrounded by a jogging track
(3 𝑚 𝑤𝑖𝑑𝑡ℎ, light shaded area). The dimensions of the park (including both
tracks) are 15𝑚 × 20𝑚. Find the area of:
A. The jogging track
B. The walking track
C. The Grass

Area of jogging track


= (15 × 20) − (9 × 14) = 300 − 126 = 174
Area of walking track
14 × 9 − 10 × 5 = 126 − 50 = 76

Example 3.10
Rectangle ABCD has sides of length 3 and 4.
A. Find the length of the diagonal AC.
B. Find the length of the perpendicular from vertex B to the
diagonal.

Similar Triangles

3.2 Trapezoids
A. Trapezoid

3.11: Definition
A quadrilateral with a pair of parallel sides is a trapezoid.

Example 3.12
𝑀𝑎𝑟𝑘 𝑎𝑙𝑙 𝑐𝑜𝑟𝑟𝑒𝑐𝑡 𝑜𝑝𝑡𝑖𝑜𝑛𝑠
Which of the following 𝑚𝑢𝑠𝑡 be trapezoids:
A. Square
B. Rectangle
C. Rhombus
D. Kite
E. Parallelogram

A parallelogram is a quadrilateral with two pairs of parallel sides.


Hence, it does have a pair of parallel sides.

P a g e 59 | 83
Get all the files at: https://bit.ly/azizhandouts
Aziz Manva (azizmanva@gmail.com)

Hence, a parallelogram is a special case of a trapezoid.

Squares, rectangles, and rhombuses are all special cases of parallelograms.


Hence, they are all trapezoids.

However, a kite is a quadrilateral that has two pairs of adjacent sides which are congruent. It may or may not
have a pair of parallel sides.
Hence, it is not necessary that a kite is a trapezoid.

Hence, the correct answer is


𝑂𝑝𝑡𝑖𝑜𝑛𝑠 𝐴, 𝐵, 𝐶, 𝐸

Example 3.13
What is the difference between a trapezoid and a trapezium?

They are the same.

3.14: Terminology
➢ The two parallel sides are called the bases.
➢ The two other sides are called the legs.
➢ The distance between the bases is called the height.

Example 3.15
The quadrilateral drawn alongside is a trapezoid.
A. Identify the lines which are parallel to each other.
B. Identify the bases, the legs and the height.

𝐿𝑖𝑛𝑒 𝐴𝐵 ∥ 𝐿𝑖𝑛𝑒 𝐶𝐷
𝐵𝑎𝑠𝑒𝑠 𝑎𝑟𝑒 𝐴𝐵 𝑎𝑛𝑑 𝐶𝐷
𝐿𝑒𝑔𝑠 𝑎𝑟𝑒 𝐴𝐷 𝑎𝑛𝑑 𝐵𝐶
𝐻𝑒𝑖𝑔ℎ𝑡 = ℎ

Example 3.167
In trapezoid 𝐴𝐵𝐶𝐷, 𝐴𝐷 is perpendicular to 𝐷𝐶, 𝐴𝐷 = 𝐴𝐵 = 3, and 𝐷𝐶 = 6. In addition,
𝐸 is on 𝐷𝐶, and 𝐵𝐸 is parallel to 𝐴𝐷. Find the area of 𝐵𝐸𝐶. (AMC 8 2007/8)

𝐵𝐸 = 𝐴𝐷 = 3
𝐸𝐶 = 𝐷𝐶 − 𝐷𝐸 = 𝐷𝐶 − 𝐴𝐵 = 6 − 3 = 3

1 1 1 9
[𝐵𝐸𝐶] = ℎ𝑏 = (𝐵𝐸)(𝐸𝐶) = ∙ 3 ∙ 3 = = 4.5 𝑢𝑛𝑖𝑡𝑠 2
2 2 2 2

3.17: Angles on the same leg are supplementary


The two angles on the same leg of a trapezoid are supplementary

In trapezoid 𝐴𝐵𝐶𝐷, let 𝐵𝐶 ∥ 𝐴𝐷. Consider 𝐴𝐵 as the transversal of 𝐵𝐶 and


𝐴𝐷.
By co-interior angles (angles on the same side of the transversal)
∠𝐶𝐵𝐴 + ∠𝐵𝐴𝐷 = 180

P a g e 60 | 83
Get all the files at: https://bit.ly/azizhandouts
Aziz Manva (azizmanva@gmail.com)

Hence, the angles are supplementary.

Example 3.18
In trapezoid 𝐴𝐵𝐶𝐷, ∠𝐴 = 50°. ∠𝐶 = 110°. Determine the measures of
∠𝐵, and ∠𝐷.

∠𝐵 = 180 − ∠𝐴 = 180 − 50° = 130°


∠𝐷 = 180 − ∠𝐶 = 180 − 110° = 70°

Example 3.19
In trapezoid 𝐴𝐵𝐶𝐷, ∠𝐴 = 40°. ∠𝐶 = ∠𝐵 + 10. Determine the
measure of ∠𝐷.

∠𝐵 = 180 − ∠𝐴 = 180 − 40 = 140°


∠𝐶 = ∠𝐵 + 10 = 140 + 10 = 150°
∠𝐷 = 180 − ∠𝐶 = 180 − 150 = 30°

Example 3.20
In the quadrilateral drawn alongside, side 𝐵𝐶 ∥ side 𝐴𝐷.
Determine the value of 𝑥 + 𝑦 given that:
∠𝐶𝐵𝐴 = 3𝑥 − 2
∠𝐵𝐶𝐷 = 3𝑦 − 7
∠𝐵𝐴𝐷 = 2𝑥 + 5
∠𝐶𝐷𝐴 = 5𝑦 + 4

177
3𝑥 − 2 + 2𝑥 + 5 = 180 ⇒ 5𝑥 + 3 = 180 ⇒ 𝑥 =
5
183
3𝑦 − 7 + 5𝑦 + 4 = 180 ⇒ 8𝑦 − 3 = 180 ⇒ 𝑦 =
8

177 183
𝑥+𝑦 = +
5 8

Example 3.21
Trapezoid 𝐴𝐵𝐶𝐷 is shown in the diagram. Determine the ratio of
∠𝐶 to ∠𝐷 given that ∠𝐴 is thirty times ∠𝐷, and that ∠𝐵 is ∠𝐴 times
∠𝐷.

Let ∠𝐷 = 𝑥
30𝑥 2 + 30𝑥 = 180
𝑥2 + 𝑥 = 6
2
𝑥 +𝑥−6 = 0
(𝑥 + 3)(𝑥 − 2) = 0
𝑥 ∈ {2, −3}
∠𝐷 = 2
∠𝐶 = 180 − 2 = 178
∠𝐶: ∠𝐷 = 178: 2 = 89: 1

P a g e 61 | 83
Get all the files at: https://bit.ly/azizhandouts
Aziz Manva (azizmanva@gmail.com)

3.22: Converse: Proving a Trapezoid


In quadrilateral 𝐴𝐵𝐶𝐷, if ∠𝐴 and ∠𝐵 are supplementary, then the
quadrilateral is a trapezoid.

Since the angles are supplementary:


∠𝐴 + ∠𝐵 = 180

But, ∠𝐴 and ∠𝐵 are co-interior angles considering AB as the transversal of lines 𝐵𝐶 and 𝐴𝐷.
∴ 𝐴𝐵 ∥ 𝐶𝐷

Hence,
𝑄𝑢𝑎𝑑𝑟𝑖𝑙𝑎𝑡𝑒𝑟𝑎𝑙 𝐴𝐵𝐶𝐷 𝑖𝑠 𝑎 𝑡𝑟𝑎𝑝𝑒𝑧𝑜𝑖𝑑
B. Perimeter

3.23: Perimeter
The sum of the lengths of the sides of a trapezoid is the perimeter.

Example 3.24
What is the perimeter of a trapezoid with bases 4 𝑐𝑚 and 6 𝑐𝑚, and legs each 2 𝑐𝑚.

4 + 6 + 2 + 2 = 14

Example 3.25
Find the perimeter of the isosceles trapezoid drawn
alongside (not to scale)
1 1 1
𝐴𝐵 = , 𝐶𝐷 = , 𝐴𝐷 =
3 2 6

1 1 1 1 2 1 3 1 7 1
𝐴𝐵 + 𝐵𝐶 + 𝐶𝐷 + 𝐴𝐷 = + + + = + + + = =1
3 6 2 6 6 6 6 6 6 6
C. Area

3.26: Area
The area of a trapezoid is given by:
𝑏1 + 𝑏2
𝐴𝑟𝑒𝑎 = ℎ ×
2

Where
ℎ = 𝐻𝑒𝑖𝑔ℎ𝑡
𝑏1 = 𝐵𝑎𝑠𝑒 1
𝑏2 = 𝐵𝑎𝑠𝑒 2

Consider Trapezoid 𝐴𝐵𝐶𝐷 (drawn alongside). Draw


➢ Height BP
➢ Diagonal DB

P a g e 62 | 83
Get all the files at: https://bit.ly/azizhandouts
Aziz Manva (azizmanva@gmail.com)

The trapezoid is made of Δ𝐴𝐵𝐷 and Δ𝐵𝐷𝐶:


1 1 1
[𝐴𝐵𝐶𝐷] = [𝐵𝐷𝐶] + [𝐴𝐵𝐷] = (𝐵𝑃)(𝐶𝐷) + (𝐵𝑃)(𝐴𝐵) = (𝐵𝑃)
⏟ (𝐶𝐷 + 𝐴𝐵)
2 2 2 𝐻𝑒𝑖𝑔ℎ𝑡

Example 3.27
Find the area of a trapezoid with height 3 𝑐𝑚, and bases 5 𝑐𝑚 and 6 𝑐𝑚?

𝑏1 + 𝑏2 5+6 11 33
×ℎ= ×3= ×3= = 16.5 cm2
2 2 2 2

Example 3.28
Find the area of a trapezoid with height 0.12 m, and bases 0. 3̅ cm and 0.16̅ cm

1
𝑏1 = 0. 3̅ = 0.333333 … = 𝑐𝑚
3
1
𝑏2 = 0.16̅ = 0.16666 … . = 𝑐𝑚
6
ℎ = 0.12 𝑚 = 12 𝑐𝑚
1 1
𝑏1 + 𝑏2 + 3 1
𝐴𝑟𝑒𝑎 = × ℎ = 3 6 × 12 = × × 12 = 3 cm2
2 2 6 2

Example 3.29
Find the area of a trapezoid with height one-third of a foot, one base five-sixth of a foot, and the other base
1
having 33 % greater length than the first base.
3

1 1
ℎ = 𝑓𝑒𝑒𝑡 = × 12 = 4 𝑖𝑛𝑐ℎ𝑒𝑠
3 3
5 5
𝑏1 = 𝑓𝑒𝑒𝑡 = × 12 = 10 𝑖𝑛𝑐ℎ𝑒𝑠
6 6

1 100 100 1 1 1 4 4 40
33 % = %= = 𝑔𝑟𝑒𝑎𝑡𝑒𝑟 ⇒ 𝑏2 = 𝑏1 + × 𝑏1 = 𝑏1 (1 + ) = 𝑏1 × = 10 × =
3 3 300 3 3 3 3 3 3

40
𝑏1 + 𝑏2 10 + 3 70 140
𝐴𝑟𝑒𝑎𝑎 = ×ℎ = ×4= ×2= inch2
2 2 3 3

D. Ratios

Example 3.30
The area of triangle 𝑋𝑌𝑍 is 8 square inches. Points 𝐴 and 𝐵 are midpoints of congruent segments 𝑋𝑌 and 𝑋𝑍.
Altitude 𝑋𝐶 bisects 𝑌𝑍. The area (in square inches) of the shaded region is (AMC 8 2002/20)

P a g e 63 | 83
Get all the files at: https://bit.ly/azizhandouts
Aziz Manva (azizmanva@gmail.com)

Example 3.31
Trapezium ABCD has AB ∥ CD and area 18.
A. AB is half the length of CD. What is the ratio of the area of △ABD to that of the trapezium? Is it possible
to answer this question without calculating the area of △ABD?
B. AB has length 6, and CD has length 8. What is Area(△ABD) : Area(△BCD)?
C. If AB has length p, and CD has length q, can you find Area(△ABD) : Area(△BCD)? Does your final
answer increase or decrease if the area of the trapezium changes.

E. Rearrangement

Example 3.32
In trapezoid 𝐴𝐵𝐶𝐷 with bases 𝐴𝐵 and 𝐶𝐷, we have 𝐴𝐵 = 52, 𝐵𝐶 = 12, 𝐶𝐷 =
39, and 𝐷𝐴 = 5 (diagram not to scale). The area of 𝐴𝐵𝐶𝐷 is (AMC 10A
2002/25)

Example 3.33
In rectangle 𝐴𝐵𝐶𝐷, 𝐴𝐵 = 6, 𝐴𝐷 = 30, and 𝐺 is the midpoint of 𝐴𝐷. Segment 𝐴𝐵 is extended 2 units beyond 𝐵 to
point 𝐸, and 𝐹 is the intersection of 𝐸𝐷 and 𝐵𝐶. What is the area of 𝐵𝐹𝐷𝐺? (AMC 10B 2012/19)

𝐺𝐷 ∥ 𝐵𝐹 ⇒ 𝐵𝐺𝐷𝐹 𝑖𝑠 𝑎 𝑡𝑟𝑎𝑝𝑒𝑧𝑜𝑖𝑑 ⇒ 𝑏1 = 𝐺𝐷 = 15, ℎ = 6


We need to find the other base, which is 𝐵𝐹.
We can prove two triangles similar by AA Similarity:
∠𝐵𝐴𝐷 = ∠𝐸𝐵𝐹 = 90°, ∠ 𝐴𝐸𝐷 = ∠𝐵𝐸𝐹 ⇒ Δ𝐵𝐸𝐹~Δ𝐴𝐸𝐷
W
𝐵𝐸 2 1 𝐵𝐹 1 30 15
= = ⇒ = ⇒ 𝐵𝐹 = =
𝐴𝐸 8 4 𝐴𝐷 4 4 2

15
15 + 2 45 135
[𝐵𝐷𝐹𝐺] = ×6= ×3 =
2 2 2

P a g e 64 | 83
Get all the files at: https://bit.ly/azizhandouts
Aziz Manva (azizmanva@gmail.com)

F. Pythagorean Theorem

Example 3.34
Quadrilateral 𝐴𝐵𝐶𝐷 is a trapezoid, 𝐴𝐷 = 15, 𝐴𝐵 = 50, 𝐵𝐶 = 20, and the
altitude is 12. What is the area of the trapezoid? (AMC 8 2011/20)

Draw 𝐷𝐺 ⊥ 𝐶𝐹 and 𝐸𝐻 ⊥ 𝐶𝐹. Since the distance between parallel


lines is the same,
𝐷𝐺 = 𝐸𝐻 = 12

By Pythagorean Triplets:
3(3,4,5) = (9,12,15) = (𝐶𝐺, 𝐷𝐺, 𝐷𝐶)
3(3,4,5) = (12,16,20) = (𝐸𝐻, 𝐻𝐹, 𝐸𝐹)
Then, the area of the trapezoid is:
[𝐷𝐶𝐺] + [𝐸𝐻𝐹] + [𝐷𝐸𝐻𝐺]
1 1
= (12)(9) + (12)(16) + (50)(12)
2 2
= 6(9) + 6(16) + (100)(6)
= 6(25) + 100
= 750

Example 3.35
What is the perimeter of trapezoid 𝐴𝐵𝐶𝐷? (AMC 8 2005/19)

By Pythagorean Triplet:
6(3,4,5) = (18,24,30) = (𝐴𝐸, 𝐵𝐸 , 𝐴𝐵) ⇒ 𝐴𝐸 = 18

By Pythagorean Triplet:
(7,24,25) = (𝑋𝐷, 𝐶𝑋, 𝐶𝐷) ⇒ 𝑋𝐷 = 7

The perimeter
= 𝐴𝐵 + 𝐵𝐶 + 𝐶𝐷 + 𝐷𝐴 = 30 + 50 + 25 + (7 + 50 + 18) = 180

Example 3.36
̅̅̅̅ and 𝐶𝐷
In trapezoid 𝐴𝐵𝐶𝐷, 𝐴𝐵 ̅̅̅̅ are perpendicular to 𝐴𝐷
̅̅̅̅, with 𝐴𝐵 + 𝐶𝐷 = 𝐵𝐶, 𝐴𝐵 < 𝐶𝐷, and 𝐴𝐷 = 7. What is
𝐴𝐵 ∙ 𝐶𝐷? (AMC 10 2001/24)

Let
𝐴𝐵 = 𝑥, 𝐶𝐷 = 𝑦
Then
𝐵𝐶 = 𝐴𝐵 + 𝐶𝐷 = 𝑥 + 𝑦, 𝐴𝐵 ∙ 𝐶𝐷 = 𝑥𝑦
Construct
𝐵𝑋 ⊥ 𝐶𝐷 ⇒ 𝐶𝑋 = 𝑦 − 𝑥

By the Pythagorean Theorem in Δ𝐵𝑋𝐶:


𝐶𝑋 2 + 𝐵𝑋 2 = 𝐵𝐶 2

P a g e 65 | 83
Get all the files at: https://bit.ly/azizhandouts
Aziz Manva (azizmanva@gmail.com)

(𝑦 − 𝑥)2 + 72 = (𝑦 + 𝑥)2

Now, we have an equation, which we can simplify:


𝑦 2 − 2𝑥𝑦 + 𝑥 2 + 49 = 𝑦 2 + 2𝑥𝑦 + 𝑥 2

Simplify and collate like terms on each side:


49 1
4𝑥𝑦 = 49 ⇒ 𝑥𝑦 = = 12
4 4

Example 3.37
The area of trapezoid 𝐴𝐵𝐶𝐷 is 164 cm2 . The altitude is 8 cm, AB
is 10 cm, and 𝐶𝐷 is 17 cm. What is BC, in centimeters? (AMC 8
2003/21)

𝐵𝐶 + 𝐴𝐷
8( ) = 164 ⇒ 𝐵𝐶 + 𝐴𝐷 = 41
2
𝐵𝐶 + 𝐵𝐶 + 6 + 15 = 41
𝐵𝐶 = 10 𝑐𝑚
G. General

Example 3.38
Points 𝑀 and 𝑁 are the midpoints of sides 𝑃𝐴 and 𝑃𝐵 of Δ𝑃𝐴𝐵. As 𝑃
moves along a line that is parallel to side 𝐴𝐵, how many of the four
quantities listed below change? (AMC 10 2000/5)
(a) the length of the segment 𝑀𝑁
(b) the perimeter of △ 𝑃𝐴𝐵
(c) the area of △ 𝑃𝐴𝐵
(d) the area of trapezoid 𝐴𝐵𝑁𝑀

Analysis
Part A
Obviously, 𝐴𝐵 does not change in length.
Therefore, by the Midpoint Theorem, MN also does not change in length.

Part B
Consider an extreme scenario. P moves far to the left (or far to the right).
The perimeter will change.

Part C
1
𝐴(△ 𝑃𝐴𝐵) = ℎ𝑏
2
Neither the height, nor the base changes, so the area does not change.

Part D

𝐴𝐵 + 𝑀𝑁
𝐴(𝑇𝑟𝑎𝑝𝑒𝑧𝑜𝑖𝑑 𝐴𝐵𝑁𝑀) = ×ℎ
2

P a g e 66 | 83
Get all the files at: https://bit.ly/azizhandouts
Aziz Manva (azizmanva@gmail.com)

None of the above quantities change, so the area does not change.

Final Answer
Out of the four given quantities, only one changes.

H. Isosceles Trapezoid

3.39: Isosceles Trapezoid


When the legs are a trapezoid are equal in length, it is an isosceles trapezoid.

Example 3.40
Decide whether each trapezoid drawn
alongside is isosceles or not.

The one on the left is not Isosceles.


The one on the right is Isosceles.

Example 3.41
A rectangular yard contains two flower beds in the shape of
congruent isosceles right triangles. The remainder of the
yard has a trapezoidal shape, as shown. The parallel sides of
the trapezoid have lengths 15 and 25 meters. What fraction
of the yard is occupied by the flower beds? (AMC 10B 2009/4)

Draw a diagram of the rectangle.


The sides of the triangle are
25 − 15 10
= =5
2 2

The right triangle can be moved to go to the left. The fraction of the yard
occupied by the flower beds is
5 1
=
25 5

3.42: Isosceles Trapezoid


➢ The base angles of an isosceles trapezium are equal
➢ The “top angles” are also equal.

Base Angles
Draw the altitudes from 𝐵 and 𝐶.
∠𝐵𝑋𝐴 = ∠𝐶𝑌𝐷 = 90° (𝑅𝑖𝑔ℎ𝑡 𝐴𝑛𝑔𝑙𝑒)

The distance between parallel lines is the same:


𝐵𝑋 = 𝐶𝑌 (𝑆𝑖𝑑𝑒)

Since the trapezium is isosceles:


𝐵𝐴 = 𝐶𝐷 (𝐻𝑦𝑝)

P a g e 67 | 83
Get all the files at: https://bit.ly/azizhandouts
Aziz Manva (azizmanva@gmail.com)

Combining the above three statements:


Δ𝐵𝑋𝐴 = 𝐶𝑌𝐷 (𝑅𝐻𝑆)
∠𝐵𝐴𝐷 = ∠𝐶𝐷𝐴 (𝐶𝑃𝐶𝑇𝐶)
Top Angles
∠𝐴𝐵𝑋 = ∠𝐷𝐶𝑌(𝐶𝑃𝐶𝑇𝐶)
∠𝐴𝐵𝑋 + 90° = ∠𝐷𝐶𝑌 + 90°
∠𝐴𝐵𝐶 = ∠𝐷𝐶𝐵

3.43: Converse: Proving an Isosceles Trapezium


If the base angles of a trapezoid are congruent, the trapezoid is isosceles.

In trapezoid ABCD draw height BX and CY.


In Δ𝐵𝐴𝑋 and Δ𝐶𝐷𝑌
∠𝐵𝐴𝑋 = ∠𝐶𝐷𝑌 (𝐺𝑖𝑣𝑒𝑛)
𝐵𝑋 = 𝐶𝑌 (𝐻𝑒𝑖𝑔ℎ𝑡 𝑖𝑠 𝑒𝑞𝑢𝑎𝑙)
∠𝐵𝑋𝐴 = ∠𝐶𝑌𝐷 = 90°(𝐷𝑒𝑓 𝑜𝑓 𝐻𝑒𝑖𝑔ℎ𝑡)
Δ𝐵𝐴𝑋 ≅ Δ𝐶𝐷𝑌 (𝑆𝐴𝐴)
𝐵𝐴 = 𝐶𝐷 (𝐶𝑃𝐶𝑇𝐶)

3.44: Diagonals of an Isosceles Trapezium


The diagonals of an isosceles trapezium are congruent.

In Δ𝐴𝐵𝐶 and Δ𝐷𝐶𝐵:


𝐴𝐵 = 𝐶𝐷 (𝐿𝑒𝑔𝑠 𝑜𝑓 𝑖𝑠𝑜𝑐𝑒𝑙𝑒𝑠 𝑡𝑟𝑎𝑝𝑒𝑧𝑖𝑢𝑚)
𝐵𝐶 = 𝐵𝐶 (𝑅𝑒𝑓𝑙𝑒𝑥𝑖𝑣𝑒 𝑃𝑟𝑜𝑝𝑒𝑟𝑡𝑦)
∠𝐴𝐵𝐶 = ∠𝐵𝐶𝐷 (𝑇𝑜𝑝 𝑎𝑛𝑔𝑙𝑒𝑠 𝑜𝑓 𝑖𝑠𝑜𝑐𝑒𝑙𝑒𝑠 𝑡𝑟𝑎𝑝𝑒𝑧𝑖𝑢𝑚)
Δ𝐴𝐵𝐶 ≅ Δ𝐷𝐶𝐵
𝐵𝐷 ≅ 𝐴𝐶 (𝐶𝑃𝐶𝑇𝐶)

3.45: Converse
If the diagonals of a trapezoid are congruent, it is an isosceles trapezoid.

Consider trapezoid ABCD with 𝐵𝐶 ∥ 𝐴𝐷. Construct 𝐶𝐸 ∥


𝐵𝐷, intersecting 𝐴𝐷 at 𝐸.

Consider 𝐵𝐷 and 𝐶𝐸 as transversals of 𝐵𝐶 and 𝐴𝐷. Since


they are corresponding angles:
∠𝐵𝐷𝐴 = ∠𝐶𝐸𝐷

𝐵𝐷 and 𝐶𝐸 are transversals to parallel lines 𝐵𝐶 and 𝐴𝐷.


Hence,𝐵𝐷 = 𝐶𝐸. Since the diagonals are congruent, 𝐴𝐶 = 𝐵𝐷. Combining the two statements:
𝐴𝐶 = 𝐶𝐸 ⇒ ΔACE is isoceles ⇒ ∠𝐶𝐴𝐷 = ∠𝐶𝐸𝐴

In Δ𝐶𝐴𝐷 and Δ𝐵𝐴𝐷:


𝐴𝐶 = 𝐴𝐶 (𝑆𝑖𝑑𝑒)
∠𝐶𝐴𝐷 = ∠𝐶𝐸𝐴 (𝐴𝑛𝑔𝑙𝑒)
𝐶𝐴 = 𝐵𝐷(𝐴𝑛𝑔𝑙𝑒) (𝐺𝑖𝑣𝑒𝑛)
Δ𝐶𝐴𝐷 ≅ Δ𝐵𝐴𝐷(𝑆𝐴𝑆 𝐶𝑜𝑛𝑔𝑟𝑢𝑒𝑛𝑐𝑒)

P a g e 68 | 83
Get all the files at: https://bit.ly/azizhandouts
Aziz Manva (azizmanva@gmail.com)

𝐴𝐵 ≅ 𝐶𝐷 (𝐶𝑃𝐶𝑇𝐶)

Example 3.46
What is the perimeter of an isosceles trapezium with height 12, shorter base 23, and longer base 32?

Longer side will have its extra length equally divided on both sides (isosceles trapezium)
Left-hand side triangle is right-angled. By Pythagoras Theorem:
Left Leg = Right Leg = √52 + 122 = √169 = 13

Perimeter = 23 + 32 + 13 * 2 = 81

Example 3.47
In trapezoid 𝐴𝐵𝐶𝐷, the sides 𝐴𝐵 and 𝐶𝐷 are equal. The perimeter of
𝐴𝐵𝐶𝐷 is (AMC 8 1999/14)

8 + 2(5) + 16 = 34 𝑢𝑛𝑖𝑡𝑠

I. Trapezoid Midsegment Theorem

3.48: Median of a Trapezoid


The median of a trapezoid is the line joining the midpoints of its two legs.

3.49: Median: Length and Parallel to Base Properties


The median of a trapezoid is parallel to its bases.

The length of the median of a trapezoid is the average of the two bases of the trapezoid.
𝐴𝐵 + 𝐶𝐷
𝑀𝑒𝑑𝑖𝑎𝑛 = 𝐸𝐹 =
2

Draw trapezoid 𝐴𝐵𝐷𝐶 with 𝐴𝐵 ∥ 𝐶𝐷.


Let 𝐸 be the midpoint of 𝐴𝐶, and 𝐹 be the midpoint of 𝐵𝐷.

Extend 𝐴𝐹 to meet 𝐶𝐷 at 𝐺.

Δ𝐴𝐵𝐹 ≅ Δ𝐷𝐹𝐺 by AAS since:


∠𝐵𝐴𝐹 = ∠𝐷𝐺𝐹 (𝐴𝑙𝑡𝑒𝑟𝑛𝑎𝑡𝑒 𝐼𝑛𝑡𝑒𝑟𝑖𝑜𝑟 𝐴𝑛𝑔𝑙𝑒𝑠)
∠𝐴𝐹𝐵 = ∠𝐷𝐹𝐺 (𝑉𝑒𝑟𝑡𝑖𝑐𝑎𝑙𝑙𝑦 𝑂𝑝𝑝𝑜𝑠𝑖𝑡𝑒 𝐴𝑛𝑔𝑙𝑒𝑠)
𝐵𝐹 = 𝐹𝐷 (𝑀𝑖𝑑𝑝𝑜𝑖𝑛𝑡 𝐷𝑒𝑓𝑖𝑛𝑖𝑡𝑖𝑜𝑛)
𝐴𝐹 = 𝐹𝐺 (𝐶𝑃𝐶𝑇) ⇒ 𝐹 𝑖𝑠 𝑚𝑖𝑑𝑝𝑜𝑖𝑛𝑡 𝑜𝑓 𝐴𝐺

By Midpoint Theorem:𝐸𝐹 ∥ 𝐶𝐷. Hence:


𝐶𝐺 𝐶𝐷 + 𝐷𝐺
𝐸𝐹 = =
2 2

P a g e 69 | 83
Get all the files at: https://bit.ly/azizhandouts
Aziz Manva (azizmanva@gmail.com)

Substitute 𝐷𝐺 = 𝐴𝐺 (𝐶𝑃𝐶𝑇 𝑖𝑛 Δ𝐴𝐵𝐹 ≅ Δ𝐷𝐹𝐺)


𝐶𝐷 + 𝐴𝐵
=
2

Example 3.50
In trapezoid 𝐴𝐵𝐶𝐷 we have 𝐴𝐵 parallel to 𝐷𝐶, 𝐸 as the midpoint of 𝐵𝐶, and 𝐹 as the midpoint of 𝐷𝐴. The area of
𝐴𝐵
𝐴𝐵𝐸𝐹 is twice the area of 𝐹𝐸𝐶𝐷. What is 𝐷𝐶? (AMC 10B 2005/23)

[𝐴𝐵𝐸𝐹] = 2[𝐹𝐸𝐶𝐷]

1
Using 𝐴𝑟𝑒𝑎 𝑜𝑓 𝑡𝑟𝑎𝑝𝑒𝑧𝑜𝑖𝑑 = 2 ℎ(𝑏1 + 𝑏2 )
1 𝑥+𝑦 1 𝑥+𝑦
ℎ [𝑥 + ( )] = 2 [ ℎ [( ) + 𝑦]]
2 2 2 2
𝑥+𝑦 𝑥+𝑦
𝑥+( ) = 2 [( ) + 𝑦]
2 2
3𝑥 + 𝑦 𝑥 + 3𝑦
= 2[ ]
2 2
3𝑥 + 𝑦 = 2𝑥 + 6𝑦
𝑥 = 5𝑦
𝐴𝐵 = 5𝐷𝐶
𝐴𝐵
=5
𝐷𝐶

3.51: Median: Midpoint of Diagonals


The median of a trapezoid passes through the midpoints of its diagonals.
In the diagram,
𝑃𝑄 𝑝𝑎𝑠𝑠𝑒𝑠 𝑡ℎ𝑟𝑜𝑢𝑔ℎ 𝑌 𝑎𝑛𝑑 𝑋
Where
𝑌 𝑖𝑠 𝑚𝑖𝑑𝑝𝑜𝑖𝑛𝑡 𝑜𝑓 𝐶𝐵
𝑋 𝑖𝑠 𝑚𝑖𝑑𝑝𝑜𝑖𝑛𝑡 𝑜𝑓 𝐴𝐷

Draw trapezoid 𝐴𝐵𝐷𝐶 with median:


𝑃𝑄 ∥ 𝐴𝐵 ∥ 𝐶𝐷

Since a parallel line divides the sides of a triangle in the same ratio:
𝐴𝑋 𝐴𝑃
= =1
𝑋𝐷 𝑃𝐶
Hence,
𝑋 𝑖𝑠 𝑡ℎ𝑒 𝑚𝑖𝑑𝑝𝑜𝑖𝑛𝑡 𝑜𝑓 𝐷𝑖𝑎𝑔𝑜𝑛𝑎𝑙 𝐴𝐷

That 𝑃𝑄 passes through the


midpoint 𝑌 of the other diagonal can be proved similarly.

3.52: Midpoint of Diagonals


P a g e 70 | 83
Get all the files at: https://bit.ly/azizhandouts
Aziz Manva (azizmanva@gmail.com)

The line joining the mid points of the diagonals of a trapezoid is equal to half of the difference of the parallel
sides.

In the diagram:
1
𝑋𝑌 = (𝐶𝐷 − 𝐴𝐵)
2

Draw trapezoid 𝐴𝐵𝐷𝐶 with midpoints of diagonals 𝑋 and


𝑌 as shown.
Let P and Q be the midpoints of 𝐴𝐶 and 𝐵𝐷 respectively.
From above, we know that the median 𝑃𝑄 passes through
𝑋 and 𝑌.

𝐶𝑜𝑛𝑠𝑡𝑟𝑢𝑐𝑡𝑖𝑜𝑛:
Extend AX till it intersects 𝐶𝐷 at F.

Δ𝐵𝑋𝐴 ≅ Δ𝐹𝑋𝐶 by ASA:


∠𝐴𝐵𝐶 = ∠𝐵𝐶𝐷 (𝐴𝑙𝑡𝑒𝑟𝑛𝑎𝑡𝑒 𝐼𝑛𝑡𝑒𝑟𝑖𝑜𝑟 𝐴𝑛𝑔𝑙𝑒𝑠)
∠𝐵𝑋𝐴 = ∠𝐹𝑋𝐶 (𝑉𝑒𝑟𝑡𝑖𝑐𝑎𝑙𝑙𝑦 𝑂𝑝𝑝𝑜𝑠𝑖𝑡𝑒 𝐴𝑛𝑔𝑙𝑒𝑠)
𝐵𝑋 = 𝑋𝐶 (𝑋 𝑖𝑠 𝑚𝑖𝑑𝑝𝑜𝑖𝑛𝑡 𝑜𝑓 𝑑𝑖𝑎𝑔𝑜𝑛𝑎𝑙 𝐵𝐶)

Hence, by CPCT:
𝐴𝑋 = 𝑋𝐹 ,
⏟ 𝐶𝐹 = 𝐴𝐵

𝑬𝒒𝒖𝒂𝒕𝒊𝒐𝒏 𝑰 𝑬𝒒𝒖𝒂𝒕𝒊𝒐𝒏 𝑰𝑰

From the first equality above, X and Y are midpoints of 𝐴𝐹 and 𝐴𝐷 respectively.
Also 𝑋𝑌 ∥ 𝐹𝐷 since 𝑃𝑄 ∥ 𝐶𝐷.
Therefore, by Midpoint Theorem:
1
𝑋𝑌 = 𝐹𝐷
2

By complementary lengths, 𝐹𝐷 = 𝐶𝐷 − 𝐶𝐹
1
𝑋𝑌 = (𝐶𝐷 − 𝐶𝐹)
2

From Equation II, 𝐶𝐹 = 𝐴𝐵


1
𝑋𝑌 = (𝐶𝐷 − 𝐴𝐵)
2
Which is what we wanted to prove.

Example 3.53
The line joining the midpoints of the diagonals of a trapezoid has length 3. If the longer base is 97, then the
shorter base is: (AHSME 1959/22)

P a g e 71 | 83
Get all the files at: https://bit.ly/azizhandouts
Aziz Manva (azizmanva@gmail.com)

1
𝑋𝑌 = (𝐶𝐷 − 𝐴𝐵)
2

Substitute 𝑋𝑌 = 3, 𝐶𝐷 = 97
1
3 = (97 − 𝐴𝐵)
2
6 = 97 − 𝐴𝐵
𝐴𝐵 = 97 − 6 = 91

3.3 Parallelograms
A. Definition

3.54: (Def) Parallelogram


A parallelogram is a quadrilateral with opposite sides parallel.

➢ Special cases of parallelograms include squares, rectangles, and rhombi.

3.55: Special Trapezoid


A parallelogram is a special case of a trapezoid, since a trapezoid has at least one pair of parallel sides, and a
parallelogram has both pairs of sides parallel.

3.56: Properties of a Trapezoid/Quadrilateral.


Since a parallelogram is a
➢ special trapezoid, it inherits all the properties of a trapezoid.
➢ special trapezoid, it inherits all the properties of a quadrilateral.

B. Angles in a Parallelogram

Example 3.57
In parallelogram 𝐴𝐵𝐶𝐷, shown alongside, identify the pairs of co-
interior angles from the interior angles of the parallelogram.

𝐴𝐵 ∥ 𝐶𝐵 hence
∠𝐵𝐴𝐶 & ∠𝐴𝐶𝐷 𝑎𝑟𝑒 𝑐𝑜 − 𝑖𝑛𝑡𝑒𝑟𝑖𝑜𝑟
∠𝐴𝐵𝐷 & ∠𝐶𝐷𝐵 𝑎𝑟𝑒 𝑐𝑜 − 𝑖𝑛𝑡𝑒𝑟𝑖𝑜𝑟

𝐴𝐶 ∥ 𝐵𝐷 hence
∠𝐵𝐴𝐶 & ∠𝐴𝐵𝐷 𝑎𝑟𝑒 𝑐𝑜 − 𝑖𝑛𝑡𝑒𝑟𝑖𝑜𝑟
∠𝐴𝐶𝐷 & ∠𝐶𝐷𝐵 𝑎𝑟𝑒 𝑐𝑜 − 𝑖𝑛𝑡𝑒𝑟𝑖𝑜𝑟

3.58: Adjacent and Opposite Angles


➢ Adjacent Angles of a parallelogram are supplementary.
➢ Opposite Angles of a parallelogram are congruent

P a g e 72 | 83
Get all the files at: https://bit.ly/azizhandouts
Aziz Manva (azizmanva@gmail.com)

Consider parallelogram 𝐴𝐵𝐶𝐷

Adjacent Angles:
Since 𝐴𝐵 ∥ 𝐶𝐷, ∠𝐶 and ∠𝐴 are co-interior angles:
∴ ∠𝐶 𝑎𝑛𝑑 ∠𝐴 𝑎𝑟𝑒 𝑠𝑢𝑝𝑝𝑙𝑒𝑚𝑒𝑛𝑡𝑎𝑟𝑦 ⇒ ∠𝐴 = 180 − ∠𝐶

We can make a similar argument for:


∠𝐶 & ∠𝐷, ∠𝐷 & ∠𝐵, ∠𝐵 & ∠𝐴

Opposite Angles:
Since 𝐴𝐷 ∥ 𝐵𝐶, ∠𝐵 and ∠𝐴 are co-interior angles:
∴ ∠𝐵 𝑎𝑛𝑑 ∠𝐴 𝑎𝑟𝑒 𝑠𝑢𝑝𝑝𝑙𝑒𝑚𝑒𝑛𝑡𝑎𝑟𝑦
∴ ∠𝐵 = 180 − ∠𝐴 = 180 − (180 − ∠𝐶) = ∠𝐶

We can make a similar argument for:


∠𝐴 & ∠𝐷

C. Height and Area

3.59: (Def) Height of a Parallelogram


The height of a parallelogram is the distance between the opposite sides.

➢ The height is often convenient to calculate when originating from a vertex, but this is not required.

Example 3.60
In parallelogram 𝐴𝐵𝐶𝐷, draw:
C. the height from 𝐴𝐵 to 𝐶𝐷
D. the height from 𝐴𝐶 to 𝐵𝐷

In parallelogram ABCD, the height from AB to CD


= 𝐷𝑖𝑠𝑡𝑎𝑛𝑐𝑒 𝑏𝑒𝑡𝑤𝑒𝑒𝑛 𝐴𝐵 𝑎𝑛𝑑 𝐶𝐷 = 𝐴𝑋 = 𝑌𝐶

The height from AD to BC is a little more difficult to visualize:


= 𝐷𝑖𝑠𝑡𝑎𝑛𝑐𝑒 𝑏𝑒𝑡𝑤𝑒𝑒𝑛 𝐴𝐷 𝑎𝑛𝑑 𝐵𝐶 = 𝐴𝐴′ = 𝐷𝐷 ′ = 𝑃𝑄

Note that
𝑃𝑄 𝑑𝑜𝑒𝑠 𝑛𝑜𝑡 𝑜𝑟𝑖𝑔𝑖𝑛𝑎𝑡𝑒 𝑓𝑟𝑜𝑚 𝑎 𝑣𝑒𝑟𝑡𝑒𝑥

P a g e 73 | 83
Get all the files at: https://bit.ly/azizhandouts
Aziz Manva (azizmanva@gmail.com)

3.61: Area of a Parallelogram


The area of a parallelogram is the product of its height and base.
𝐴 = ℎ𝑏

Method I: Congruence of Triangles


The diagonal of a parallelogram divides it into two congruent triangles.
Hence:
1
[𝐴𝐵𝐶𝐷] = [𝐴𝐷𝐶] + [𝐴𝐵𝐶] = 2(𝐴Δ𝐴𝐷𝐶) = 2 ( ℎ𝑏) = (𝐴𝐸)(𝐷𝐶)
2

Method II: Comparing to a Rectangle


In parallelogram 𝐴𝐵𝐶𝐷, create rectangle 𝐴𝐵𝐹𝐸 by drawing
𝐴𝐸 ⊥ 𝐶𝐷, 𝐵𝐹 ⊥ 𝐶𝐷

Δ𝐷𝐴𝐸 ≅ Δ𝐶𝐵𝐹 by 𝐴𝐴𝑆:


𝐷𝐴 = 𝐶𝐵 (𝑂𝑝𝑝. 𝑠𝑖𝑑𝑒𝑠 𝑜𝑓 ∥ −𝑔𝑟𝑎𝑚)
∠𝐴𝐷𝐸 = ∠𝐵𝐶𝐹 (𝐶𝑜𝑟𝑟𝑒𝑠𝑝𝑜𝑛𝑑𝑖𝑛𝑔 𝐴𝑛𝑔𝑙𝑒𝑠)
∠𝐴𝐸𝐷 = ∠𝐵𝐹𝐶 = 90°

Hence:
[𝐴𝐵𝐶𝐷] = [𝐴𝐵𝐶𝐸] + [𝐷𝐴𝐸] = [𝐴𝐵𝐶𝐸] + [𝐶𝐵𝐹] = [𝐴𝐵𝐹𝐸] = 𝐴𝐵 ∙ 𝐴𝐸

Method III: Cutting and Moving


In parallelogram 𝐴𝐵𝐶𝐷, draw
𝐴𝐸 ⊥ 𝐶𝐷

Translate Δ𝐴𝐷𝐸 right till 𝐴𝐷 is located over 𝐵𝐶, giving:


Δ𝐵𝐶𝐹

𝐴𝐸 ⊥ 𝐶𝐷 ⇒ 𝐵𝐹 ⊥ 𝐶𝐷 ⇒ 𝐴𝐵𝐶𝐹 𝑖𝑠 𝑎 𝑟𝑒𝑐𝑡𝑎𝑛𝑔𝑙𝑒

Hence:
[𝐴𝐵𝐶𝐷] = [𝐴𝐵𝐹𝐸] = 𝐴𝐵 ∙ 𝐴𝐸

Method IV: Special Case of Trapezoid


A parallelogram is a special case of a trapezoid. Hence, use the formula for area of a trapezoid with 𝑏 = 𝑏1 = 𝑏2
𝑏1 + 𝑏2 𝑏+𝑏
𝐴= ℎ= ℎ = ℎ𝑏
2 2
D. Diagonals

Example 3.62
Diagonal AC is drawn in parallelogram 𝐴𝐵𝐶𝐷. Identify pairs of congruent angles. Identify pairs of congruent
angles.

P a g e 74 | 83
Get all the files at: https://bit.ly/azizhandouts
Aziz Manva (azizmanva@gmail.com)

Since the opposite angles of a parallelogram are congruent:


∠𝐴𝐵𝐶 = ∠𝐴𝐷𝐶, ∠𝐵𝐴𝐶 = ∠𝐷𝐶𝐵 (𝑂𝑝𝑝. 𝑎𝑛𝑔𝑙𝑒𝑠 𝑜𝑓 ∥ −𝑔𝑟𝑎𝑚)

Consider parallel lines 𝐴𝐷 and BC. AC is the transversal. Hence:


∠𝐷𝐴𝐶 = ∠𝐴𝐶𝐵 (𝐴𝑙𝑡𝑒𝑟𝑛𝑎𝑡𝑒 𝐼𝑛𝑡𝑒𝑟𝑖𝑜𝑟 𝐴𝑛𝑔𝑙𝑒𝑠)

Consider parallel lines 𝐴𝐵 and CD. AC is the transversal. Hence:


∠𝐴𝐶𝐷 = ∠𝐵𝐴𝐶 (𝐴𝑙𝑡𝑒𝑟𝑛𝑎𝑡𝑒 𝐼𝑛𝑡𝑒𝑟𝑖𝑜𝑟 𝐴𝑛𝑔𝑙𝑒𝑠)

3.63: Triangles formed by Diagonals / Opp sides are ≅


➢ A diagonal of a parallelogram divides it into two congruent triangles.
➢ Hence, opposite sides of a parallelogram are congruent.

Δ𝐴𝐵𝐶 ≅ Δ𝐴𝐷𝐶 by ASA:


∠𝐷𝐴𝐶 = ∠𝐴𝐶𝐵, ∠𝐴𝐶𝐷 = ∠𝐵𝐴𝐶 (𝑃𝑟𝑒𝑣𝑖𝑜𝑢𝑠 𝐸𝑥𝑎𝑚𝑝𝑙𝑒)
𝐴𝐶 = 𝐴𝐶 (𝑅𝑒𝑓𝑙𝑒𝑥𝑖𝑣𝑒)
𝐴𝐵 = 𝐶𝐷 (𝐶𝑃𝐶𝑇)

Similarly, we can show


𝐴𝐷 = 𝐵𝐶 𝑖𝑛 ≅ Δ′ 𝑠 𝐴𝐷𝐵 & 𝐶𝐷𝐵

3.64: Diagonals Bisect


The diagonals of a parallelogram bisect each other.

That is, if 𝑋 is the intersection point of the diagonals, then:


𝑋 𝑖𝑠 𝑡ℎ𝑒 𝑚𝑖𝑑𝑝𝑜𝑖𝑛𝑡 𝑜𝑓 𝐴𝐶
𝑋 𝑖𝑠 𝑡ℎ𝑒 𝑚𝑖𝑑𝑝𝑜𝑖𝑛𝑡 𝑜𝑓 𝐵𝐷

In parallelogram 𝐴𝐵𝐶𝐷:
Δ𝐴𝑋𝐷 ≅ Δ𝐵𝑋𝐶 by ASA:
𝐴𝐷 = 𝐵𝐶 (𝑂𝑝𝑝 𝑠𝑖𝑑𝑒𝑠 𝑜𝑓 𝑎 ∥ 𝑔𝑟𝑎𝑚)
𝐴𝐷 ∥ 𝐵𝐶 ⇒ 𝐴𝐶 𝑖𝑠 𝑡𝑟𝑎𝑛𝑠𝑣𝑒𝑟𝑠𝑎𝑙 ⇒ 𝐴𝑙𝑡𝑒𝑟𝑛𝑎𝑡𝑒 𝐼𝑛𝑡𝑒𝑟𝑖𝑜𝑟: ∠𝐷𝐴𝐶 = ∠𝐴𝐶𝐵
𝐴𝐵 ∥ 𝐶𝐷 ⇒ 𝐴𝐶 𝑖𝑠 𝑡𝑟𝑎𝑛𝑠𝑣𝑒𝑟𝑠𝑎𝑙 ⇒ 𝐴𝑙𝑡𝑒𝑟𝑛𝑎𝑡𝑒 𝐼𝑛𝑡𝑒𝑟𝑖𝑜𝑟: ∠𝐴𝐷𝐵 = ∠𝐷𝐵𝐶
𝑋𝐷 = 𝑋𝐵, 𝑋𝐴 = 𝑋𝐶 (CPCT)

E. Proving a Quadrilateral to be a Parallelogram

3.65: Opposite Sides are parallel


If the opposite sides of a quadrilateral are parallel, the quadrilateral is a parallelogram.

This is the definition of a parallelogram.

3.66: Opposite Angles are congruent


If the opposite angles of a quadrilateral are congruent, then the quadrilateral is a parallelogram.

In quadrilateral 𝐴𝐵𝐶𝐷, if
∠𝐴 = ∠𝐶, ∠𝐵 = ∠𝐷 ⇒ 𝐴𝐵𝐶𝐷 𝑖𝑠 𝑎 𝑝𝑎𝑟𝑎𝑙𝑙𝑒𝑙𝑜𝑔𝑟𝑎𝑚

P a g e 75 | 83
Get all the files at: https://bit.ly/azizhandouts
Aziz Manva (azizmanva@gmail.com)

Given: Opposite angles of a quadrilateral are congruent.


To be proved: The quadrilateral is a parallelogram.

Draw quadrilateral 𝐴𝐵𝐶𝐷 with opposite angles


congruent.

Step I: Establish a relation between the angles


By the sum of the angles of a quadrilateral:
2𝑥 + 2𝑦 = 360°
𝑥 + 𝑦 = 180°

Step II: Show that 𝑨𝑪 ∥ 𝑩𝑫


∠𝐵𝐴𝐷 + ∠𝐴𝐷𝐶 = 180° ⇒ 𝐴𝑛𝑔𝑙𝑒𝑠 𝑎𝑟𝑒 𝑠𝑢𝑝𝑝𝑙𝑒𝑚𝑒𝑛𝑡𝑎𝑟𝑦

∠𝐵𝐴𝐶 and ∠𝐴𝐷𝐶 are co-interior angles with transversal 𝐴𝐷 of lines 𝐴𝐵 and 𝐶𝐷. If co-interior angles are
supplementary, then the lines are parallel:
∴ 𝐴𝐶 ∥ 𝐵𝐷

Step III: Show that 𝑨𝑫 ∥ 𝑩𝑪


∠𝐷𝐴𝐵 + ∠𝐴𝐵𝐶 = 180° ⇒ 𝐴𝑛𝑔𝑙𝑒𝑠 𝑎𝑟𝑒 𝑠𝑢𝑝𝑝𝑙𝑒𝑚𝑒𝑛𝑡𝑎𝑟𝑦

∠𝐷𝐴𝐵 and ∠ABC are co-interior angles with transversal 𝐴𝐵 of lines 𝐴𝐷 and 𝐵𝐶. If co-interior angles are
supplementary, then the lines are parallel:
∴ 𝐴𝐷 ∥ 𝐵𝐶

Since the quadrilateral has both pairs of sides parallel, by definition


𝐼𝑡 𝑖𝑠 𝑎 𝑝𝑎𝑟𝑎𝑙𝑙𝑒𝑙𝑜𝑔𝑟𝑎𝑚

3.67: Diagonals Bisect


If the diagonals of a quadrilateral bisect each other, the quadrilateral is
a parallelogram.

That is, in quadrilateral 𝐴𝐵𝐶𝐷, if 𝑋 is the intersection point of the


diagonals and
𝑋𝐴 = 𝑋𝐵, 𝑋𝐵 = 𝑋𝐷 ⇒ 𝐴𝐵𝐶𝐷 𝑖𝑠 𝑎 𝑝𝑎𝑟𝑎𝑙𝑙𝑒𝑙𝑜𝑔𝑟𝑎𝑚

Given: The diagonals of a quadrilateral bisect each other.


To be proved: The quadrilateral is a parallelogram.

In quadrilateral 𝐴𝐵𝐶𝐷:
Δ𝐴𝑋𝐷 ≅ Δ𝐵𝑋𝐶 by SAS:
𝑋𝐷 = 𝑋𝐵, 𝑋𝐴 = 𝑋𝐶 (𝐺𝑖𝑣𝑒𝑛)
∠𝐴𝑋𝐷 = ∠𝐵𝑋𝐶 (𝑉𝑒𝑟𝑡𝑖𝑐𝑎𝑙𝑙𝑦 𝑜𝑝𝑝𝑜𝑠𝑖𝑡𝑒 𝑎𝑛𝑔𝑙𝑒𝑠)
𝐴𝐷 = 𝐵𝐶 (𝐶𝑃𝐶𝑇)

Δ𝐴𝑋𝐵 ≅ ΔD𝑋𝐶 by SAS:


𝑋𝐷 = 𝑋𝐵, 𝑋𝐴 = 𝑋𝐶 (𝐺𝑖𝑣𝑒𝑛)
∠𝐴𝑋𝐵 = ∠𝐷𝑋𝐶 (𝑉𝑒𝑟𝑡𝑖𝑐𝑎𝑙𝑙𝑦 𝑜𝑝𝑝𝑜𝑠𝑖𝑡𝑒 𝑎𝑛𝑔𝑙𝑒𝑠)

P a g e 76 | 83
Get all the files at: https://bit.ly/azizhandouts
Aziz Manva (azizmanva@gmail.com)

𝐴𝐵 = 𝐷𝐶 (𝐶𝑃𝐶𝑇)

Hence, the opposite sides of quadrilateral 𝐴𝐵𝐶𝐷 are congruent.


Hence, the quadrilateral is a parallelogram.

3.68: Opposite Sides are congruent


If the opposite sides of a quadrilateral are congruent, the quadrilateral is a parallelogram.

That is, in quadrilateral 𝐴𝐵𝐶𝐷


𝐴𝐷 = 𝐵𝐶, 𝐴𝐵 = 𝐶𝐷 ⇒ 𝐴𝐵𝐶𝐷 𝑖𝑠 𝑎 𝑝𝑎𝑟𝑎𝑙𝑙𝑒𝑙𝑜𝑔𝑟𝑎𝑚

Given: Opposite sides of a quadrilateral are equal


To be proved: Quadrilateral is a parallelogram

Δ𝐴𝐵𝐶 ≅ Δ𝐴𝐷𝐶 by SSS:


𝐴𝐷 = 𝐵𝐶, 𝐴𝐵 = 𝐶𝐷 (𝐺𝑖𝑣𝑒𝑛)
𝐴𝐶 = 𝐴𝐶 (𝑅𝑒𝑓𝑙𝑒𝑥𝑖𝑣𝑒)
∠𝐷𝐴𝐶 = ∠𝐴𝐵𝐶, ∠𝐷𝐶𝐴 = ∠𝐵𝐴𝐶 (𝐶𝑃𝐶𝑇)

Consider AC as transversal of AD and CB:


𝐴𝑙𝑡𝑒𝑟𝑛𝑎𝑡𝑒 𝐼𝑛𝑡𝑒𝑟𝑖𝑜𝑟 𝐴𝑛𝑔𝑙𝑒𝑠 (𝐷𝐴𝐶 & 𝐴𝐶𝐵) 𝑎𝑟𝑒 ≅⇒ 𝐴𝐷 ∥ 𝐶𝐵

Consider AC as transversal of AB and CD:


𝐴𝑙𝑡𝑒𝑟𝑛𝑎𝑡𝑒 𝐼𝑛𝑡𝑒𝑟𝑖𝑜𝑟 𝐴𝑛𝑔𝑙𝑒𝑠 (𝐷𝐶𝐴 & 𝐵𝐴𝐶) 𝑎𝑟𝑒 ≅⇒ 𝐴𝐵 ∥ 𝐶𝐷

In Quadrilateral ABCD:
𝐴𝐷 ∥ 𝐶𝐵, 𝐴𝐵 ∥ 𝐶𝐷 ⇒ 𝑂𝑝𝑝. 𝑆𝑖𝑑𝑒𝑠 𝑎𝑟𝑒 𝑝𝑎𝑟𝑎𝑙𝑙𝑒𝑙 ⇒ 𝑃𝑎𝑟𝑎𝑙𝑙𝑒𝑙𝑜𝑔𝑟𝑎𝑚

3.69: One pair of opposite sides is congruent and parallel


If one pair of opposite sides of a quadrilateral is congruent and parallel, the quadrilateral is a parallelogram.

That is, in quadrilateral 𝐴𝐵𝐶𝐷


𝐴𝐵 = 𝐶𝐷 & 𝐴𝐵 ∥ 𝐶𝐷 ⇒ 𝐴𝐵𝐶𝐷 𝑖𝑠 𝑎 𝑝𝑎𝑟𝑎𝑙𝑙𝑒𝑙𝑜𝑔𝑟𝑎𝑚

Given: One pair of opposite sides of a quadrilateral is congruent and


parallel.
To be proved: The quadrilateral is a parallelogram.

Δ𝐴𝐵𝐶 ≅ Δ𝐴𝐷𝐶 by SAS:


𝐴𝐵 = 𝐶𝐷 (𝐺𝑖𝑣𝑒𝑛)
∠𝐵𝐴𝐶 = ∠𝐷𝐶𝐴 (𝐴𝑙𝑡𝑒𝑟𝑛𝑎𝑡𝑒 𝐼𝑛𝑡𝑒𝑟𝑖𝑜𝑟 𝐴𝑛𝑔𝑙𝑒𝑠)
𝐴𝐶 = 𝐴𝐶 (𝑅𝑒𝑓𝑙𝑒𝑥𝑖𝑣𝑒)

Hence, in the given quadrilateral:


𝐴𝐵 = 𝐶𝐷 (𝐺𝑖𝑣𝑒𝑛), 𝐴𝐷 = 𝐵𝐶 (𝐶𝑃𝐶𝑇)

Hence, both pairs of opposite sides are congruent.


As shown above, if both pairs of opposite sides are congruent, then the quadrilateral is a parallelogram.

P a g e 77 | 83
Get all the files at: https://bit.ly/azizhandouts
Aziz Manva (azizmanva@gmail.com)

F. Joining Midpoints
We can use the methods that we have learnt of proving a parallelogram to prove further properties of
parallelograms.

3.70: Joining Midpoints gives a Parallelogram


The quadrilateral formed by joining the midpoints of a parallelogram 𝑖𝑛 𝑜𝑟𝑑𝑒𝑟 is also a parallelogram.

That is, in parallelogram 𝐴𝐵𝐶𝐷 join the midpoints to get quadrilateral 𝑃𝑄𝑅𝑆. Then:
𝑃𝑄𝑅𝑆 𝑖𝑠 𝑎 𝑝𝑎𝑟𝑎𝑙𝑙𝑒𝑙𝑜𝑔𝑟𝑎𝑚

Given: A quadrilateral is formed by joining the midpoints of a parallelogram.


To be proved: The quadrilateral by joining the midpoints is a parallelogram.

Draw diagonal 𝐷𝐵.


𝑃 is midpoint of 𝐴𝐷, and 𝑄 is midpoint of 𝐴𝐵. Hence, by
Midpoint Theorem, in Δ𝐴𝐷𝐵L
1
𝑃𝑄 ∥ 𝐷𝐵, 𝑃𝑄 = 𝐷𝐵
⏟ 2
𝑹𝒆𝒔𝒖𝒍𝒕 𝟏

𝑅 is midpoint of 𝐵𝐶, and 𝑆 is midpoint of 𝐶𝐷. Hence, by


Midpoint Theorem, in Δ𝐶𝐵𝐷:
1
𝑆𝑅 ∥ 𝐷𝐵, 𝑆𝑅 = 𝐷𝐵
⏟ 2
𝑹𝒆𝒔𝒖𝒍𝒕 𝟐

From Result 1 and 2:


𝑃𝑄 ∥ 𝑆𝑅,
⏟ 𝑃𝑄 = 𝑆𝑅
𝑹𝒆𝒔𝒖𝒍𝒕 𝟑

If one pair of opposite sides in a quadrilateral is parallel and congruent, the quadrilateral is a parallelogram.
Hence:
𝑃𝑄𝑅𝑆 𝑖𝑠 𝑎 𝑝𝑎𝑟𝑎𝑙𝑙𝑒𝑙𝑜𝑔𝑟𝑎𝑚
G. Constructing a Parallelogram

Example 3.71
Quadrilateral 𝐴𝐵𝐶𝐷 has 𝐴𝐵 = 𝐵𝐶 = 𝐶𝐷, angle 𝐴𝐵𝐶 = 70 and angle 𝐵𝐶𝐷 = 170. What is the measure of angle
𝐵𝐴𝐷? (AMC 10B 2008/24)

P a g e 78 | 83
Get all the files at: https://bit.ly/azizhandouts
Aziz Manva (azizmanva@gmail.com)

Draw
➢ Diagonal DB
➢ Line DE congruent to DC and parallel to AB to create parallelogram ABED

In Isosceles 𝚫𝑫𝑪𝑩
180 − ∠𝐷𝐶𝐵 180 − 170
∠𝐶𝐷𝐵 = ∠𝐶𝐵𝐷 = = =5
2 2
∴ ∠ 𝐴𝐵𝐷 = ∠𝐴𝐵𝐶 − ∠𝐶𝐵𝐷 = 70 − 5 = 65

In parallelogram 𝑨𝑩𝑬𝑫:
∠𝐵𝐷𝐸 = 65 (𝐴𝑙𝑡𝑒𝑟𝑛𝑎𝑡𝑒 𝐼𝑛𝑡𝑒𝑟𝑖𝑜𝑟 𝐴𝑛𝑔𝑙𝑒𝑠)
∠𝐸𝐷𝐶 = ∠𝐵𝐷𝐸 − ∠𝐵𝐷𝐶 = 65 − 5 = 60

In Isosceles 𝚫𝑫𝑪𝑩
∠𝐸𝐷𝐶 = 60 ⇒ ∠𝐷𝐸𝐶 = ∠𝐷𝐶𝐸 = 60° ⇒ 𝛥𝐸𝐶𝐷 𝑖𝑠 𝑒𝑞𝑢𝑖𝑙𝑎𝑡𝑒𝑟𝑎𝑙.

In parallelogram 𝑨𝑩𝑬𝑫, using parallelogram properties:


∠𝐵𝐴𝐷 = ∠𝐷𝐸𝐵 = 𝑥
∠𝐶𝐸𝐵 = 𝑥 − 60
∠𝐴𝐵𝐸 = 180 − 𝑥
∠𝐶𝐵𝐸 = ∠𝐴𝐵𝐸 − ∠𝐴𝐵𝐶 = 110 − 𝑥

Since 𝚫𝑬𝑪𝑩 is isosceles:


𝑥 − 60 = 110 − 𝑥 ⇒ 𝑥 = 85

3.4 Rhombi
A. Definition

3.72: Equilateral Quadrilateral


➢ A rhombus is an 𝑒𝑞𝑢𝑖𝑙𝑎𝑡𝑒𝑟𝑎𝑙 quadrilateral.
➢ Equilateral means all the sides are equal.

➢ Plural of rhombus is rhombuses or rhombi

3.73: A rhombus is also a parallelogram/kite


➢ Every rhombus is a parallelogram.
➢ Every rhombus is a parallelogram.

Step I: Show that a rhombus is a parallelogram.


In Rhombus 𝐴𝐵𝐶𝐷
𝐴𝐵 = 𝐶𝐷, 𝐴𝐷 = 𝐵𝐶

Since the opposite sides of the quadrilateral are equal:


𝐼𝑡 𝑖𝑠 𝑎 𝑝𝑎𝑟𝑎𝑙𝑙𝑒𝑙𝑜𝑔𝑟𝑎𝑚

Step I: Show that a rhombus is a kite.


𝐴𝐵 = 𝐴𝐷, 𝐶𝐵 = 𝐶𝐷

Since two pairs of adjacent sides are equal, by definition:


P a g e 79 | 83
Get all the files at: https://bit.ly/azizhandouts
Aziz Manva (azizmanva@gmail.com)

𝐼𝑡 𝑖𝑠 𝑎 𝐾𝑖𝑡𝑒

Example 3.74
𝑀𝑎𝑟𝑘 𝑎𝑙𝑙 𝑐𝑜𝑟𝑟𝑒𝑐𝑡 𝑜𝑝𝑡𝑖𝑜𝑛𝑠
It is true that every:
A. rhombus is a parallelogram.
B. parallelogram is a rhombus.
C. rhombus is a quadrilateral
D. rhombus is a trapezoid
E. trapezoid is a rhombus

𝑂𝑝𝑡𝑖𝑜𝑛𝑠 𝐴, 𝐶, 𝐷
B. Diagonals

1 Pending

3.75: Diagonals of a Rhombus


The diagonals of a rhombus are perpendicular to each other.

2 Pending

3.76: Diagonals of a Rhombus


The diagonals of a rhombus bisect the vertex angles.

3 Pending

3.77: Diagonals of a Rhombus


The diagonals of a rhombus divide it into four congruent triangles.

4 Pending

3.78: Diagonals of a Rhombus


The diagonals of a rhombus are perpendicular bisectors of each other

C. Proving a Rhombus from a Parallelogram

5 Pending

3.79: Parallelogram with perpendicular diagonals


If the diagonals of a parallelogram are perpendicular to each other, the parallelogram is a rhombus.

P a g e 80 | 83
Get all the files at: https://bit.ly/azizhandouts
Aziz Manva (azizmanva@gmail.com)

6 Pending

3.80: Parallelogram with Diagonal that bisects a vertex angle


If the diagonal of a parallelogram bisects a vertex angle, the parallelogram is a rhombus.

D. Midpoints

7 Pending

3.81: Midpoints form a Rectangle


The quadrilateral formed by joining the midpoints of the sides of a rhombus in order form a rectangle. And this
rectangle has area half of the rhombus.

3.5 General Quadrilaterals


A. Angles in a Quadrilateral

3.82: Sum of Angles


The sum of angles of a quadrilateral is
360°

A quadrilateral can be divided into two triangles which each have sum 180°.
Hence, the total of the angles of a quadrilateral is
180 × 2 = 360°

Example 3.83
The angles of a quadrilateral are in the ratio 1: 2: 3: 4. Determine the values of the angles.

1 + 2 + 3 + 4 = 10
360
= 36
10
36 × 1 = 36
36 × 2 = 72
36 × 3 = 108
36 × 4 = 144

Example 3.84
The largest angle in a quadrilateral is 30° more than the smallest angle. The other angles each are right angles.
Determine the values of the angles.

Suppose the smallest angle is 𝑥. Then the largest is 𝑥 + 30


𝑥 + 90 + 90 + 𝑥 + 30 = 360
2𝑥 = 360 − 210
2𝑥 = 150
150
𝑥= = 75
2

P a g e 81 | 83
Get all the files at: https://bit.ly/azizhandouts
Aziz Manva (azizmanva@gmail.com)

𝑥 + 30 = 105

{75,90,90,105}

B. Perimeter

Example 3.85
A. What is the perimeter (in feet) of a regular pentagon with side 6 inches?
B. What is the perimeter (in inches) of a regular pentagon with side 6 feet?
C. A pentagonal field with each side five meters is to be fenced using fencing that costs five dollars per
meter. Fenceposts that cost five dollars each are to be placed at a distance one meter apart. What is the
total cost?

Side * 6 = 6 * 6 = 36 inches = 3 feet


Side * 5 = 6 * 5 = 30 feet = 360 inches feet
Cost = Fencing + Fenceposts = 5 * 5 * 5 + 5 * 5 * 5 = 125 + 125 = 250

C. Triangle Inequality

3.86: Triangle Inequality


Sum of lengths of any two sides of a triangle is greater than the third side.

Example 3.87
In quadrilateral 𝐴𝐵𝐶𝐷, 𝐴𝐵 = 5, 𝐵𝐶 = 17, 𝐶𝐷 = 5, 𝐷𝐴 =
9, and 𝐵𝐷 is an integer. What is 𝐵𝐷? (AMC 10A
2009/12; AMC 12A 2009/10)

In Δ𝐴𝐷𝐵, by the triangle inequality:


𝐵𝐷 < 𝐷𝐴 + 𝐴𝐵 = 9 + 5 = 14
𝐵𝐷 < 14

𝑰𝒏𝒆𝒒𝒖𝒂𝒍𝒊𝒕𝒚 𝑰

In Δ𝐶𝐷𝐵, by the triangle inequality:


𝐵𝐷 + 𝐶𝐷 > 𝐵𝐶
𝐵𝐷 + 5 > 17
𝐵𝐷 > 12

𝑰𝒏𝒆𝒒𝒖𝒂𝒍𝒊𝒕𝒚 𝑰𝑰
Combining Inequalities I and II:
12 < 𝐵𝐷 < 14
Since BD is an integer:
𝐵𝐷 = 13
D. Quadrilateral Inequality

3.88: Quadrilateral Inequality


Sum of lengths of any three sides of a quadrilateral is greater than the fourth side

P a g e 82 | 83
Get all the files at: https://bit.ly/azizhandouts
Aziz Manva (azizmanva@gmail.com)

Example 3.89
Joy has 30 thin rods, one each of every integer length from 1 cm through 30 cm. She places the rods with lengths
3 cm, 7 cm, and 15 cm on a table. She then wants to choose a fourth rod that she can put with these three to
form a quadrilateral with positive area. How many of the remaining rods can she choose as the fourth rod?
(AMC 10A 2017/10)

Since the quadrilateral must have positive area, it cannot be a degenerate quadrilateral. That is, the rods should
form a “proper quadrilateral.”

Let the length of the fourth rod be 𝑟. By the quadrilateral inequality:


𝑟 < 3 + 7 + 15
𝑟 < 25

The longest(given) side must be less than the sum of the other three sides:
15 < 𝑟 + 3 + 7
15 < 𝑟 + 10
𝑟>5

5 < 𝑟 < 25
{6, 𝟕, 8,9,10,11,12,13,14, 𝟏𝟓, 16,17,18,19,20,21,22,23,24} ⇒ 19 𝑟𝑜𝑑𝑠

Remove 7 and 15 since those rods have already been used:


19 − 2 = 17 𝑟𝑜𝑑𝑠

3.6 Further Topics


90 Examples

P a g e 83 | 83

You might also like